You are on page 1of 107

免费领取GMAT资料、了解GMAT课程,可加微信:jmtccca

Sentence Correction

819
The market for so-called functional beverages, drinks that promise health benefits beyond their
inherent nutritional value, nearly doubled over the course of four years, in rising from $2.68
billion in 1997 to be $4.7 billion in 2000.
A. in rising from $2. 68 billion in 1997 to be ----from to be 搭配不对,in rising 语义不符合表达
习惯
B. in having risen from $2. 68 billion In 1997 to ---in having 同样不符合表达习惯
C. as it rose from $2.68 billion in 1997 to be---from to be,as 在这里语义不对
D. with its rise from $2.68 billion in 1997 to----its 多余,with 这里做介词后面不能跟句子,语
义不对,带有它从这里到那里
E. rising from $2. 68 billion in 1997 to---svo,doing 状语,固定搭配 from to。所谓功能性饮料的
市场,即承诺健康益处超过其固有营养价值的饮料,在四年中几乎翻了一番,从 1997 年的
26.8 亿$上升到 2000 年的 47 亿$。

820.
According to some critics, watching television not only undermines one’s ability to think critically
but also impairs one's overall ability to perceive.
A. not only undermines one’s ability to think critically but also impairs one’s.
B. not only undermines one’s ability of critical thinking but also impairs the.
C. undermines not only one's ability to think critically but also impairs one’s.
D .undermines not only one’s ability of critical thinking but also impairs the
E.undermines one’s ability not only to think critically but also impairs one’s
解析:选 A
句子主干 watching television not only undermines..to think ...but also impairs...to perceive。
原文逻辑为看电视不仅会削弱人批判性思考的能力,而且还会损害人整体感知的能力。原
文逻辑正确,不轻易改变,且 not only 和 but also 前后平行。
B 项 undermines 后面是 one’s ability of,而 impairs 后面是 the overall ability to,不平行,且
ability of 表达的是“...的”,属性性质,而 ability to 是有能力去做某事,含义有差别。
C、D、E 项同样是 not only 和 but also 前后不平行
考点:平行

821.
To show that it is serious about addressing the state's power crisis, the administration has plans for
ordering all federal facilities in California to keep thermostats at 78 degrees Fahrenheit and
shutting down escalators during electricity shortages this summer.

A. has plans for ordering all federal facilities in California to keep thermostats at 78 degrees
Fahrenheit and shutting--TO do 优于 for doing, and 前后不平行,shutting 没词并列,语义没
问题

免费加入GMAT备考群、机经群,可加微信:jmtccca
免费领取GMAT资料、了解GMAT课程,可加微信:jmtccca

B. has plans to order that all federal facilities in California are keeping thermostats at 78 degrees
Fahrenheit and shutting---时态不对,不是 are keeping,plan to order 表达累赘,设备不能去
shutting,俩并列不对
C. is planning on ordering all federal facilities in California to keep thermostats at 78 degrees
Fahrenheit, and they will shut---is planning on ordering 和 to keep 句 式 杂 糅 , they 指 代
adminstration,有点啰嗦
D. is planning to order that all federal facilities in California are keeping thermostats at
78 degrees Fahrenheit and shut--时态问题,然后 are keeping 和 shut 不并列
E. is planning to order all federal facilities in California to keep thermostats at 78 degrees
Fahrenheit and shut 为了表明这是解决该州电力危机的一个严肃问题,政府计划命令加州的
所有联邦设施将恒温器保持在华氏 78 度,并在今年夏天电力短缺时关闭自动扶梯。

822.
Once made exclusively from the wool of sheep that roam the Isle of Lewis and Harris off the coast
of Scotland, Harris tweed 花呢套装 is now made only with wools that are imported, sometimes
from the mainland and sometimes they come—as a result of a 1996 amendment to the
Harris Tweed Act—from outside Scotland.

A. sometimes from the mainland and sometimes they come---they 无指代,and 前后不平行。
B. sometimes from the mainland and sometimes--平行。 哈里斯花呢曾经是专门用在苏格兰海
岸的刘易斯岛和哈里斯岛上的羊毛制成的,现在只用从苏格兰境外进口的羊毛制成,有时是
从大陆进口的,有时是-由于 1996 年对《哈里斯花呢法》的修正案-从苏格兰境外进口的。
C. and come sometimes from the mainland or sometimes----and 连接两个动词,这个 and 是连接
imported 还是 is 呢,有歧义
D. from the mainland sometimes, or sometimes it comes---谁 from the mainland?It 无指代。
E. from the mainland sometimes, or sometimes coming--or 前后不平行。

823.
In her presentation, the head of the Better Business Bureau emphasized that companies should
think of the cost of conventions and other similar gatherings as not an expense, but as an
investment in networking that will pay dividends.

A.as not an expense, but as

B.as not expense but

C.not an expense, rather

D.not as an expense, but as

免费加入GMAT备考群、机经群,可加微信:jmtccca
免费领取GMAT资料、了解GMAT课程,可加微信:jmtccca

E.not in terms of expense, but

A. not...but 前后不平行,前后概念对等,结构对等,平行无歧义。
B. not...but 前后不平行
C. Not 和 but 搭配,rather 不对
D. 正确
E. not...but 前后不平行

824.
Recent interdisciplinary studies advance the argument that emotions, including those deemed
personal or private is a social phenomenon, though one inseparable from bodily response.

A.private is a social phenomenon, though one inseparable

B.private, are social phenomena that are inseparable

C.private are a social phenomenon but are not those separable

D.private—are social phenomena but not separable

E.also as private emotions, are social phenomena not inseparable

正确答案是 B
A.原句主句可以找出是 studies advance the argument,即研究提出一个争论,没问题,但是,
原句 that 引导的同位语从句句法结构错误,emotions 后面没有谓语动词。
C:选项 C 和原句一样的错误,使得同位语从句中无主句 并且 but not those separable 无平
行对象。
D:单个破折号后面表解释说明,无主语,且 but not separable 无平行对象。
E:or also as private emotions 无法找到平行对象。not inseparable 不能作为 phenomena 的定
于后置,不符合表达习惯。
B:including 一直到 private 做插入语成分,拿掉之后就是 emotions are social phenomena,后
面再 that 的定语从句修饰 phenomena,没错,句子结构完整,也没有其他或者逻辑上的错误,
正确。

825.
In a speech before the Senate Banking Committee, the chairman of the Federal Reserve
painted an optimistic picture of the economy, suggesting to investors the central bank in the
near future is not lowering interest rates.

A.suggesting to investors the central bank in the near future is not lowering interest rates

B.suggesting to investors that the central bank would not lower interest rates in the near
future

免费加入GMAT备考群、机经群,可加微信:jmtccca
免费领取GMAT资料、了解GMAT课程,可加微信:jmtccca

C.which suggests that to investors in the near future interest rates will not be lowered by the
central bank

D.with the suggestion to investors in the near future that interest rates would not be lowered
by the central bank

E.with the suggestion to investors of interest rates not being lowered in the near future by
the central bank

解析:选 B
主干是:chairman painted an picture ,suggesting to investors the bank is not lowering interest
rates。原句首先错在 suggesting 后应该有一个 that 表示暗示的内容,而不是直接 suggesting
to investors 后面又来一个动词 is,且前面时态是过去式,这里 is not lowering 时态错误。状
语 suggesting 表伴随可以,句意是 主席描绘了一个画面,同时暗示了央行在未来将不会下
调利率,B 正确,在过去描绘的画面,预示将来会怎样,过去将来时。
C 项就近修饰 economy,逻辑错。过去将来时用 would。
D 项动词变为名词 suggestion,that 歧义修饰 future,逻辑变。
E 项同 D, not being lowered 错

826.
The company's CEO backed away from her plan for dividing the firm into five parts, saying that
she still had meant to spin off or sell two units but that the company would retain ownership of
two others as well as the core company.

A. plan for dividing the firm into five parts, saying that she still had meant--过去完成时,没必
要,to do 优于 for doing,原文状语关系逻辑对,改下措辞即可。
B. plan that was to divide the firm into five parts, and she said that she still would mean---that 修
饰 plan 不简洁,原文的状语关系逻辑对,就不轻易改成 and 关系,and 并列的前后内容没有
状语和主句关系筋膜。Would mean 时态多余。
C. plan to divide the firm into five parts, saying that she still meant 正确。该公司的首席执行官
放弃了她将公司分成五个部分的计划,说她仍然打算分拆或出售两个单位,但该公司将保留
其他两家公司以及核心公司的所有权。
D. planning on dividing the firm into five parts, and saying that she meant still----and 平 行 了
backed 和 saying,谓语动词不能和现在分词平行,并且 and 前后句式也不平行。
E. planning to divide the firm into five parts, and she said that she meant still--错误同 B,planning
有自己名词形式用自己名词形式

827.
The cactus is now heavily plundered in deserts in the southwestern United States, so much that
enforcement agencies in five states have created special squads for its protection.

A. The cactus is now heavily plundered in deserts in the southwestern United States, so much that
enforcement agencies in five states have created special squads for its protection.-----so much that
用法错,这里是 its,指代模糊

免费加入GMAT备考群、机经群,可加微信:jmtccca
免费领取GMAT资料、了解GMAT课程,可加微信:jmtccca

B.The cactus is now heavily plundered in deserts in the southwestern United States, so much so
that special squads have been created by law enforcement agencies in five states for protecting
them.------so much so that 用法也不对,这里是 them,指代模糊,so much so that 用法错。
C.The cactus is now so heavily plundered in deserts in the southwestern United States that
enforcement agencies in five states have created special squads to protect it.仙人掌现在在美国西
南部的沙漠中被如此严重的掠夺,五个州的执法机构已经建立了专门的小队来保护它。It
指仙人掌。
D.Because they are now so heavily plundered in deserts in the southwestern United States,
enforcement agencies in five states have created special squads to protect cacti.---从句主语要和
主句主语一致,they 指代 enforcement agencies 语义错。原文没有因果关系,人为添加。
E. Because they are now so heavily plundered in the southwestern United States, special squads
have been created by enforcement agencies in five states for the protection of cacti.因为仙人掌被
破坏的很严重,所以特殊小组已经被这个部门所创立,逻辑不对,时态错误。 they 指代和
从句保持一致,语义还是不对

828.
Satellite radio transmissions, a popular feature in car stereos, differ from those of AM and FM
radio, which is sent directly from earthbound towers and then to a car stereo.

A. which is sent directly from earthbound towers and then to a car---from to 固定搭配前后要对
等,中间 and then 多余
B. which are sent directly from earthbound towers to a car's 卫星无线电传输是汽车音响中的
一种流行功能,它不同于 AM 和 FM 收音机,它们直接从地面发射塔发送到汽车立体声。
which 按照语义来说,应该是修饰 AM FM,
C. sent from earthbound towers and then directly to a car----逗号链接俩动词,differ,sent,then
多余。
D. sending them directly from earthbound towers to a car's---做状语语义不对,them 代词多余
E. being sent directly from earthbound towers to a car--being 不简洁,作状语逻辑错。

829.
Although the company's executives have admitted that there had been accounting irregularities
involving improper reporting of revenue, as well as of failure to record expenses, they could not
yet say precisely how much money was involved.

A. the company's executives have admitted that there had been accounting irregularities involving
improper reporting of revenue, as well as of failure to record------as w ell as 不是连词前后平行前
面 of,后面 to 也不对等,语义能理解。--AS WELL AS 不是连词,从属的意思,这里应该
是 2 个违规项,而不是平行 reporting of 和后面的 of,语义不对。
B. the company's executives admitted that there had been accounting irregularities involving
improper reporting of revenue and failure to record--involving 现 在 分 词 短 语 后 置 修 饰
irregularities, 该公司的高管承认,存在会计违规行为,涉及不当的收入报告和未记录费用,
他们还不能确切地说涉及多少钱。所以是 improper reporting 和 failure 两个内容的平行。

免费加入GMAT备考群、机经群,可加微信:jmtccca
免费领取GMAT资料、了解GMAT课程,可加微信:jmtccca

C. the company's executives, admitting accounting irregularities involving improper reporting of


revenue and failure in recording---缺谓语
D. Admission 承认坦白 by the company's executives was made of accounting irregularities 会计
违 规 行 为 involving improper reporting of revenue and failure in recording--- 从 句 配 主 句
admission 和主句的 they,语义不对等。语义中心变,不是 admission was made of
E. admission by the company's ex
F. ecutives that there had been accounting irregularities involving improper reporting of revenue
and failure in recording---句式残缺,admission 后缺谓语

830.
A.as with 不表比较,其后主句主语 structure 不可比, reproducing 和未划线部分 serve 不平

B.to reproduce 和未划线部分 serve 不平行
C.as with 使用错,including 作状语不优,for reproducing 和未划线部分 serve 不平行
D.正确
E.that of 多余,that 在 SC 题目里不修饰人,including 状语没必要。
831.
While Noble Sissle may be best known for his collaboration with Eubie Blake, as both a
vaudeville performer and as a lyricist for songs and Broadway musicals, also enjoying an
independent career as a singer with such groups as Hahn’s Jubilee Singers.

A.and as a lyricist for songs and Broadway musicals, also enjoying

B.and writing lyrics for songs and Broadway musicals, also enjoying

C.and a lyricist for songs and Broadway musicals, he also enjoyed

D.as well as writing lyrics for songs and Broadway musicals, he also enjoyed

E.as well as a lyricist for songs and Broadway musicals, he had also enjoyed

题干想要表达的意思是也许 Noble Sissle 最知名的还是和 Eubie Blakea 的合作,Noble Sissle

既是一位杂耍演员,也是一位歌曲及百老汇音乐的作曲人,他也非常享受歌手这样一个事业,

享受有像 Hahn’s Jubilee 等歌手这样的团队。

首先,根据 both...and 平行结构排除 D,E

A. 无主句,排除,且 as 部分我们讲就近修饰的话是指 Eubie Blakea,而实际要表达的是 Noble

Sissle,所以会有歧义。

免费加入GMAT备考群、机经群,可加微信:jmtccca
免费领取GMAT资料、了解GMAT课程,可加微信:jmtccca

B. Both...and 部分不平行,排除。另外,无主句,排除。

C. 正确 both and 内容平行,且有主句,he 也有明确指代是 Noble Sissle,正确。

832.
Air traffic routes over the North Pole are currently used by only two or three planes a day, but
it was found by a joint Canadian-Russian study to be both feasible as well as desirable if those
routes are opened to thousands more commercial planes a year.

A.Air traffic routes over the North Pole are currently used by only two or three planes a day, but it
was found by a joint Canadian-Russian study to be both feasible as well as desirable if those
routes are opened to thousands more commercial planes a year.

B.Currently used by only two or three planes a day, a joint Canadian-Russian study has found
that if air traffic routes over the North Pole are opened to thousands more commercial planes a
year, it would be both feasible and desirable.

C.A joint Canadian-Russian study, finding it to be both feasible as well as desirable to open air
traffic routes over the North Pole, which are currently used by only two or three planes a day, to
thousands more commercial planes a year.

D.Although air traffic routes over the North Pole are currently used by only two or three planes a
day, a joint Canadian-Russian study has found that opening those routes to thousands more
commercial planes a year is both feasible and desirable.

E.With air traffic routes over the North Pole currently used by only two or three planes a day,
opening those routes to thousands more commercial planes a year has been found by a joint
Canadian-Russian study as both feasible and desirable.

A.首先原句逻辑没错,但是原句 both...and 的固定搭配用错,排除。题干意思是北极地区的

交通运输线路目前每天只有 2-3 架飞机会使用,但是,一个有加拿大及俄罗斯共同合作的研

究表明,如果这些线路每年能供千万架商业飞机使用的话,这将会是非常值得且可行的。

根据逻辑角度可以排除 B,C,E 所以,正确答案是 D。

B:失去原文的转折逻辑,并且,study 发不出 use 的动作,排除。

C:逻辑意思改变,且无谓语动词。Both and 搭配不对。并且给人的感觉是 study 发出的 find

的动作,不对,排除。

E,opening those routes to thousands more commercial planes a year has been found by a joint

免费加入GMAT备考群、机经群,可加微信:jmtccca
免费领取GMAT资料、了解GMAT课程,可加微信:jmtccca

Canadian 不符合表达习惯,排除,且失去原文的转折关系,排除。

833.
From an experiment using special extrasensory perception cards, each bearing one of a set of
symbols, parapsychologist Joseph Banks Rhine claimed statistical proof for subjects who
could use thought transference to identify a card in the dealer’s hand.

A.for subjects who could use thought transference to identify a card in the dealer’s hand

B.for a card in the dealer’s hand to be identified by subjects with thought transference

C.of subjects able to identify with thought transference a card in the dealer’s hand

D.that subjects could identify a card in the dealer’s hand by using thought transference

E.that subjects are capable to use thought transference for identifying a card in the dealer’s
hand

A、B、C:同位语 proof that 优选,比介词短语要清晰。然后原句逻辑不太对,应该强调的

是可以判断出卡片,而不是说使用思想转移来判断卡片。根据同位语从句优选,排除 A,B,C

D:可以发现 could 由于 able,且选项 D 的逻辑更合理,强调的是通过思想转移(这种方式)

来判断卡片,

E:强调的是能够使用思想转移,并且选项 be capable to 搭配错,正确的表达应该是 be capable

of。

834.
A long-term study of some 1,000 physicians indicates that the more coffee these doctors
drank, the more they had a likelihood of coronary disease.

A.more they had a likelihood of coronary disease

B.more was their likelihood of having coronary disease

C.more they would have a likelihood to have coronary disease

免费加入GMAT备考群、机经群,可加微信:jmtccca
免费领取GMAT资料、了解GMAT课程,可加微信:jmtccca

D.greater was their likelihood of having coronary disease

E.greater was coronary disease likely

A,B,C:因为 more 是 much many 的比较级,表示的是数量,而可能性是一个程度的问题,

所以 greater 的修饰更合理些。原文语义没有 D 清晰,A 的重点在医生咖啡喝得越多,他们

越有可能有一个可能性

D:正确,倒装,the more..the greater 比较级正确,意思是医生咖啡喝得越多,他们得冠心

病得可能性越大

E:逻辑不对,不是医生咖啡喝得越多,就越有可能是冠心病

835.
Hurricanes at first begin traveling from east to west, because that direction is the way the
prevailing winds in the tropics blow, but they then veer off toward higher latitudes, in many
cases changing direction toward the east before dissipating over the colder, more northerly
waters or over land.

A.Hurricanes at first begin traveling from east to west, because that direction is the way the
prevailing winds in the tropics blow, but

B.At first, hurricanes travel from east to west, because that is the direction of the prevailing
winds in the tropics, but

C.While hurricanes travel from east to west at first, the direction of the prevailing winds
blowing in the tropics, and

D.Because hurricanes at first travel from east to west, since it is the direction of the
prevailing winds in the tropics,

E.Hurricanes, beginning by traveling from east to west, because this is the direction of the
prevailing winds in the tropics,

A:首先,原句 at first 和 begin 逻辑意思重复,且 the direction is the way 逻辑意思不对,排

除。

C,D,E 改变了原句的转折逻辑,排除。

C,while 的句子没有主句,只有 while 一个从句,排除。

E,hurricanes 找不到谓语动词,排除。

免费加入GMAT备考群、机经群,可加微信:jmtccca
免费领取GMAT资料、了解GMAT课程,可加微信:jmtccca

836.
Travelers from Earth to Mars would have to endure low levels of gravity for long periods of
time, avoiding large doses of radiation, plus contending with the chemically reactive Martian
soil, and perhaps even ward off contamination by Martian life-forms.

A.Mars would have to endure low levels of gravity for long periods of time, avoiding large
doses of radiation, plus contending

B.Mars would have to endure low levels of gravity for long periods of time, avoid large
doses of radiation, contend

C.Mars, having to endure low levels of gravity for long periods of time, would also have to
avoid large doses of radiation, plus contending

D.Mars, having to endure low levels of gravity for long periods of time, avoid large doses of
radiation, plus contend

E.Mars, who would have to endure low levels of gravity for long periods of time avoid large
doses of radiation, contend with

A:主干是 Travelers would have to endure ..., avoiding ..., plus contending .., and ward off.逻辑

为:旅行者从地球到火星将不得不忍受低重力,避免辐射,加上与化学反应的竞争,甚至可

能阻止..的污染。原文 4 个动作平行,逻辑合理,但语法错误,plus 不是连词,未划线部分

的 ward off 和前面的 contending,avoiding,endure 不平行。

B 正确。endure,avoid,content,ward off 平行。

C 跳过 having to endure,谓语接的是 would also have to avoid ,改变逻辑,且和后面的 ward

off 不平行。

D 同 C,跳过 having to,谓语改变,plus 不是连词。

E 缺谓语,改变逻辑。

837.
Unlike the virginal,whose single set of strings runs parallel to the front edge of the instrument,
the harpsichord's several sets of strings are placed at right angles to its front edge.

免费加入GMAT备考群、机经群,可加微信:jmtccca
免费领取GMAT资料、了解GMAT课程,可加微信:jmtccca

A.whose single set of strings runs parallel to the front edge of the instrument,the
harpsichord's several sets of strings are

B.with a single set of strings running parallel to the front edge of the instrument,the several
sets of strings of the harpsichord are

C.which has a single set of strings that runs parallel to the front edge of the instrument, in
the case of the harpsichord, several sets of strings are

D.which has a single set of strings that run parallel to the front edge of the instrument, the
harpsichord has several sets of strings

E.in which a single set of strings run parallel to the front edge of the instrument,the
harpsichord's several sets of strings are

A:virginal 和 the harpsichord's several sets of strings 不可比,排除。

B,the virginal 和 the several sets of strings of the harpsichord 不可比,排除。

C,the virginal 和 the several sets of strings of the harpsichord 不可比,排除。

D,正确。run 和 strings 保持主谓一致还是和 a set of 保持主谓一致其实都能讲的通,这题

具体看语义。

E,the virginal 和 the harpsichord's several sets of strings 不可比,set 和 run 主谓搭配不一致,

排除。

838.
Many population studies have linked a high-salt diet to high rates of hypertension 高度紧张 and
shown that in societies where they consume little salt, their blood pressure typically does not rise
with age.

A. shown that in societies where they consume little salt, their ---their 无指代,they 做主语也无
指代。
B. shown that in societies that have consumed little salt, their---their 无指代,完成时没必要
C. shown that in societies where little salt is consumed, 许多人口研究将高盐饮食与高高血
压联系起来,并表明在少盐摄入的社会中,血压通常不会随着年龄的增长而升高。一般来
讲 where 修饰具体地点名词,这题用 where 修饰了抽象的 societies 只能按照大结构大语义
来理解。
D. showing that in societies where little salt is consumed,---------and 前后不平行,其他错误同上
E. showing that in societies where they consume little salt, their------and 前后不平行

免费加入GMAT备考群、机经群,可加微信:jmtccca
免费领取GMAT资料、了解GMAT课程,可加微信:jmtccca

,其他错误同上

839.
Partly as a result of Proposition 13, which severely limits property tax increases, California has
come more and more to rely on personal income taxes for its revenues; income taxes have
increased from 18 percent of the state's general fund in 1962 to 48 percent by 2002.

A. have increased from 18 percent of the state's general fund in 1962 to 48 percent by 2002---逻
辑没错,时态错,by 2002 后面可以跟过去完成时,如果要跟过去时,要看整个句子语义和
时态。increased 不是一个延续性持续性动作,不能用完成时。
B. have increased from 18 percent of the state's general fund in 1962 to being up to 48 percent by
2002-being 不简洁,就是 from to
C. have increased from 18 percent of the state's general fund in 1962, and by 2002 were up to
48 percent---and 前后不平行,完成时错。
D. increased from 18 percent of the state's general fund in 1962, and by 2002 were
48 percent---and 前后不平行
E. increased from 18 percent of the state's general fund in 1962 to 48 percent by 2002-----过去的
时间,1962 和 2002,一般过去式即可。
所得税从 1962 年占国家普通基金的 18%增加到 2002 年的 48%

840.
According to scientists, human expansion and the human appropriation of Earth's finite resources
is the cause of what may be the most sweeping wave of species extinctions since the demise of the
dinosaurs 65 million years ago.

A. expansion and the human appropriation of Earth's finite resources is---主谓不一致


B. expansion and human appropriation of Earth's finite resources are 根据科学家的说法,人类
扩张和人类占有地球有限资源是自 6500 万年前恐龙灭绝以来物种灭绝最广泛的原因。
C. expansion and its appropriation of Earth's finite resources is-----its 无指代,主谓不一致
D. expansion, along with their appropriation of Earth's finite resources, is---their 无指代,主谓不
一致,along with 啰嗦。
E. expansion, along with its appropriation of Earth's finite resources, are---along with 多余,主语
是 human expansion 也没必要用 are

841.
Although Alice Walker published a number of essays, poetry collections, and stories during
the 1970s, her third novel, The Color Purple, which was published in 1982, brought her the
widest acclaim in that it won both the National Book Award as well as the Pulitzer Prize.

A.which was published in 1982, brought her the widest acclaim in that it won both the
National Book Award as well as the Pulitzer Prize

B.published in 1982, bringing her the widest acclaim by winning both the National Book

免费加入GMAT备考群、机经群,可加微信:jmtccca
免费领取GMAT资料、了解GMAT课程,可加微信:jmtccca

Award and the Pulitzer Prize

C.published in 1982, brought her the widest acclaim, winning both the National Book Award
and the Pulitzer Prize

D.was published in 1982 and which, winning both the National Book Award and the Pulitzer
Prize, brought her the widest acclaim

E.was published in 1982, winning both the National Book Award as well as the Pulitzer
Prize, and bringing her the widest acclaim

A.原句逻辑意思没问题,意思是虽然 AW 出版了很多文章,但他的第三部小说给她带来了

广泛称赞。In that 不优选这里不是因果关系,both and 固定搭配。

B,主句无谓语动词,排除。By 改变原文逻辑。

C,正确

D,Which brought 无平行对象,且将原句的主干谓语成分变为修饰,逻辑变。Both ..and 固

定搭配

E,语义重心不对:虽然 AW 出版了很多文章但她的第三部小说出版于 1982。winning 状语

逻辑不对,both..and 不平行。

842.
Heating oil and natural gas futures rose sharply yesterday, as long-term forecasts for much
colder temperatures in key heating regions raised fears of insufficient supplies capable of
meeting the demand this winter.

A.of insufficient supplies capable of meeting

B.of supplies that would be insufficient for meeting

C.of insufficient supplies that are unable to meet

D.that there would be supplies insufficient for meeting

E.that supplies would be insufficient to meet

免费加入GMAT备考群、机经群,可加微信:jmtccca
免费领取GMAT资料、了解GMAT课程,可加微信:jmtccca

A:句 子主 干: Heating oil and natural gas futures rose sharply, as forecasts raised fears of
insufficient supplies。原句逻辑是油和天然气上升的很厉害,是因为预测引起了对供应不足
能够在今年冬天满足需求的担忧。 既然都是不充足的供给了,怎么还能满足需求呢?原文
逻辑不对。所以不应该用 of 所有格来修饰这个 fears,应该用同位语从句 that 去修饰 fears 的
具体内容或既定事实,而不是 of 表示“...的”,属性,capable of doing 冗杂了,应该是去满
足今年冬天的需求,而不是不充足的供给有能力满足冬天需求,排除 A,B,C
D 项 there 多余,修饰不简洁,to do 优于 for doing,表目的去满足需求。
E 项争取,引起了担忧,啥担忧?担忧供给将会不足以去满足今年冬天的需求。

843.
Because it regarded the environmentalists as members of an out-of-state organization, the city
council voted that they are denied permission for participating in the parade

A.that they are denied permission for participating

B.that they be denied permission for participating

C.denying them permission for participation

D.the denial of permission that they participate

E.to deny them permission to participate

A:原文正确逻辑应该是:因为它把环保主义者视为一个外州组织的成员,这个 council 投票
去否决他们,禁止他们参加。不选A,B,不用 that 来表示,voted that 表示 vote 的内容
是啥,而这里的逻辑应该是他们 vote 去否决,不让他们参加游行。
B里的原形be动词是正确的,表示命令要求请求建议的词,从句动词用原型。但有其他
错,不是为了 participating他们被 denied。逻辑有问题
C,D项里 denial 的用法错误。可以是 vote to deny;vote for denying,但不能是 vote denying。
E 项正确。To do 优于 for doing,vote 去否决他们, 让他们禁止参加

844.
In 1913, the largely self- taught Indian mathematician Srinivasa Ramanujan mailed 120 of his
theorems to three different British mathematicians;only one, G. H. Hardy, recognized the
brilliance of these theorems, but thanks to Hardy’s recognition, Ramanujan was eventually
elected to the Royal Society of London.

A.only one, G. H. Hardy, recognized the brilliance of these theorems, but

B.they were brilliant, G. H. Hardy alone recognized, but

C.these theorems were brilliant, but only one, G. H. Hardy recognized;

免费加入GMAT备考群、机经群,可加微信:jmtccca
免费领取GMAT资料、了解GMAT课程,可加微信:jmtccca

D.but, only one, G. H. Hardy, recognizing their brilliance,

E.only one G. H. Hardy recognized, but these theorems were brilliant

A :One 指的是 GH 这个人,原文的 but 是递进关系,注意积累 OG 里对 but 递进关系的用

B, they 指代不明,排除。

C,one 可以指 theorems 也可以指 GH 这个人,有歧义。

D,无谓语动词,排除。

E,brilliant 后面句式错误。

845.
Cost cutting and restructuring has allowed the manufacturing company to lower its projected
losses for the second quarter, and they are forecasting a profit before the end of the year.

A.has allowed the manufacturing company to lower its projected losses for the second
quarter, and they are forecasting

B.has allowed for the manufacturing company to lower its projected losses in the second
quarter and to forecast

C.have allowed that the manufacturing company can lower the projected losses for the
second quarter, and to forecast

D.have allowed the manufacturing company to lower its projected second quarter losses and
to forecast

E.have allowed for the manufacturing company to lower the projected losses in the second
quarter, as well as forecasting

A.主语是 Cost cutting and restructuring,谓语动词要用复数,排除 A,B 其次代词 they 指代 Cost

cutting and restructuring 不对。

C,and to forecast 无平行对象,排除。

D,正确。成本削减和重组使得制造业公司降低其第二季度的预期成本,并且在今年年底希

望实现一个盈利。

E,allow for 不符合表达习惯,且 as well as 的表达没有 and 好,没有侧重在 forecast 这个动

免费加入GMAT备考群、机经群,可加微信:jmtccca
免费领取GMAT资料、了解GMAT课程,可加微信:jmtccca

作上。排除。

846.
The Life and Casualty C o m p a n y hopes that by increasing its environmental fund reserves
to $1.2 billion,that it has set aside enough to pay for environmental claims and no longer
has to use its profits and capital to pay those claims bit by bit, year by year.

A.that it has set aside enough to pay for environmental claims and no longer has

B.enough has been set aside with which environmental claims can be paid and it will have
no longer

C.it has set aside enough for payment of environmental claims and thus no longer having

D.enough has been set aside to pay for environmental claims, thus no longer having

E.it has set aside enough to pay for environmental claims and will no longer have

A.宾语从句中无主句,排除。

B,主语变,逻辑变,with which 修饰问题,以及代词 it 指代。

C,And having 无平行对象,排除。

D,主语变,逻辑变,且 thus no longer having 变成一个状语,修饰就近句子,逻辑意思改

变,改变原句的 I 个并列逻辑,排除。

E,正确。It 指代 company,并列后主语一致,省略主语 it。

847.
Like ancient Egyptian architectural materials that were recycled in the construction of ancient
Greek Alexandria, so ancient Greek materials from the construction of that city were reused in
subsequent centuries by Roman, Muslim, and modern builders.

A.Like ancient Egyptian architectural materials that were recycled in the construction of

B.Like recycling ancient Egyptian architectural materials to construct

C.Just as ancient Egyptian architectural materials were recycled in the construction of

D.Just as they recycled ancient Egyptian architectural materials in constructing

免费加入GMAT备考群、机经群,可加微信:jmtccca
免费领取GMAT资料、了解GMAT课程,可加微信:jmtccca

E.Just like ancient Egyptian architectural materials that were recycled in constructing

A.like 后面对象和后面句子里的结构不可比
B,ancient Egyptian architectural materials 和 recycling ancient Egyptian architectural materials

不可比,且 recycling ancient Egyptian architectural materials 和后面的 were 主谓搭配不一致,

排除。

C,Just as A do,so do B,正确。

D,They 从逻辑意思上应该是指人,但是这里 they 的指代不清,且与后面的内容不平行。

排除。

E,与原文一样的问题,无 like...so 搭配错,排除。

848.
Especially in the early years, new entrepreneurs may need to find resourceful ways, like
renting temporary office space or using answering services, that make their company seem
large and more firmly established than they m a y actually be.

A.that make their comp any seem large

B.to make their companies seem larger

C.thus making their companies seem larger

D.so that the companies seem larger

E.of making their company seem large

A.原句插入语拿掉,that 修饰 ways,逻辑上不对,另外,large 和 and more firmly established

不平行

B,to do 不定式表目的,正确。

C,状语修饰就近句子,逻辑上表示新企业需要找到一些有策略的方式,因此使得公司看起

来更大更规模化,逻辑意思不对。

D,So that 逻辑关系不对,这里不是一个强烈因果关系。

E,选项 E 除了 large 的不平行问题,同时语法上,their company 和 seem 主谓搭配不一致。

免费加入GMAT备考群、机经群,可加微信:jmtccca
免费领取GMAT资料、了解GMAT课程,可加微信:jmtccca

排除。

849.
Unlike the nests of leaf cutters and most other ants, situated underground or in pieces of wood,
raider ants make a portable nest by entwining their long legs to form “curtains” of ants that
hang from logs or boulders, providing protection for the queen and the colony larvae and
pupae.

A.the nests of leaf cutters and most other ants,

B.the nests of leaf cutters and most other ants, which are

C.leaf cutters and most other ants, whose nests are

D.leaf cutters and most other ants in having nests

E.those of leaf cutters and most other ants with nests

A.原句 the nests 和 raider ants 不可比 ,由此可排除 A,B

C 正确.WHOS 定语从句后面名词从属于前面名词,正确。

D.In having nests 用法错,不放在名词后定语后置。

E.Those 指代不明,比较对象自然也不对等。

850.
Turtles, like other reptiles, can endure long fasts, in their ability to survive on weekly or even
monthly feedings; however, when food is readily available, they may eat frequently and grow
very fat.

A.fasts, in their ability to survive

B.fasts, having their ability to survive

C.fasts, due to having the ability of surviving

D.fasts because they are able to survive

E.fasts because of having the ability of surviving

免费加入GMAT备考群、机经群,可加微信:jmtccca
免费领取GMAT资料、了解GMAT课程,可加微信:jmtccca

A.in their ability to 表达 awkward,排除。

B,having 在这里作状语逻辑不对,一般不用实义动词 have 去作状语。

C,Due to 在 SC 里不表示因为,优先用 because。

D,正确

E,Because of 在句子改错中不优选,且 ability of 在这题里不优选,表达的是去生存的能力,

强调动作,所以 ability of 不优。

851.

Thai village crafts, as with other cultures, have developed through the principle that form

follows function and incorporate readily available materials fashioned using traditional skills.

A.as with

B.as did those of

C.as they have in

D.like in

E.like those of

A.crafts 和 other culture 不可比。且 as with 不表示比较,排除。

B,不可比,排除。

C,They 完全指代指代 Thai village crafts,逻辑错,但是助动词 have 正确因为整个句子时态

是完成时。

D,对象无法比较,排除。

E,正确。Those 指代 crafts.

852.
With near to all tortilla chips made from corn kernels that have been heated in a
solution of calcium hydroxide (lime), this removes the skin of the kernel so water can
penetrate.

免费加入GMAT备考群、机经群,可加微信:jmtccca
免费领取GMAT资料、了解GMAT课程,可加微信:jmtccca

A. With near to all tortilla chips made from corn kernels that have been heated in a
solution of calcium hydroxide (lime), this--this 无指代
B. Having nearly all tortilla chips made from corn kernels that are heated in a
solution of calcium hydroxide (lime), this--this 无指代,Having 用法不对
C. Nearly all tortilla chips being made from corn kernels that are heated in a solution
of calcium hydroxide (lime)----being 不 简洁 , 语 义 中 心 变 , 变 成 chips 谓 语去
removes
D. Nearly all tortilla chips are made from corn kernels that have been heated in a
solution of calcium hydroxide (lime), a process that----句子同位正确。几乎所有的玉
米饼屑都是由在氢氧化钙(石灰)溶液中加热的玉米粒制成的,这个过程这会去
除内核的皮肤,这样水就可以渗透。
E. Nearly all tortilla chips are made from corn kernels having been heated in a
solution of calcium hydroxide (lime), a process that----having been 修饰不对

853.
A.主干是:To estimate the rate is a problem because there is a lack of a single yardstick。逻辑
是:为了估算这个 rate 是一个困难,因为这里缺少一个所有距离能被测量的单一的标准,
逻辑有点混乱是吧,原句 to estimate 用的不好,直接表示一个动作开头用动名词开头即可。
B 项正确,Estimating the is a problem,因为 there is no single yardstick by which all distances
can be measured , by which 调 过 来 就 是 all distances can be measured by which
(which=yardstick),逻辑正确。估算宇宙的膨胀率是一个众所周知的难题,因为没有一
个单一的标准可以测量所有的距离。
C 项先把原因放在了前面,逗号+doing 是状语,改变了句子逻辑。By 后面无搭配。
D 项逻辑也不对,一个困难是去估算这个 rate,because 的后面 is lacking 后直接接 by which
错误,短语是 be lack of。
E 项逻辑同样不对,且不简洁很绕,说去估算这个 rate 是一个问题,because 后面直接接了
by no single yardstick 句式杂糅。

854.
Although the earliest inhabitants of Mapungubwe, building their dwellings of either
wattle and daub or unfired mud brick, by the thirteenth century buildings of coral
blocks in lime mortar began to appear.

A. earliest inhabitants of Mapungubwe, building their dwellings------inhabitants 后面


没有谓语
B. earliest inhabitants of Mapungubwe, who built their dwellings----同 A,WHO 跳跃
修饰,但最好还是不要这样,会有歧义
C. earliest inhabitants of Mapungubwe built their dwellings 虽然马普布韦最早的居
民建造他们的住所,无论是荆条和涂布或未点燃的泥砖,但到 13 世纪,珊瑚块
的建筑开始出现在石灰砂浆。
D. dwellings 住处 of the earliest inhabitants of Mapungubwe, built----重心变,不是
住处 build,主谓之间不同加逗号

免费加入GMAT备考群、机经群,可加微信:jmtccca
免费领取GMAT资料、了解GMAT课程,可加微信:jmtccca

E. dwellings of the earliest inhabitants of Mapungubwe, which were built---which 就


近修饰也不好,dwellings 后面没有谓语动词

855.

The Commerce Department reported that the nations economy grew at a brisk annual pace of

3.7 percent in the second quarter, but that while businesses were expanding their

production, unsold goods piled up on store shelves as consumer spending is slowed sharply.

A.unsold goods piled up on store shelves as consumer spending is slowed sharply

B.unsold goods were piling up on store shelves as consumer spending slowed sharply

C.unsold goods had piled up on store shelves with a sharp slowing of consumer Spending

D.consumer spending was slowing sharply, with the piling up of unsold goods on Store

shelves

E.consumer spending has slowed sharply, with unsold goods piling up on store shelves

A. 主 干 是 the Department reported that ...., but that while businesses were expanding their
production, unsold goods piled up ...as consumer spending is slowed sharply.逻辑为企业正在扩
大生产,随着消费者支出大幅放缓的同时,未售出的商品堆积如山。但 A 项的 piled up 不
能作谓语,while 后面用法错误,as 相当于 when,后面 is slowed 时态错误,前面都是过去
式,这里来一个现在时。
B 项正确,while businesses were expanding,unsold goods were piling up AS ...,while 用法正
确,表示与此同时
C 项没有过去的过去,后面 with 接的是一个名词,逻辑错:未售出的商品堆积如山,带有
一个顾客消费的缓慢?
D 项逻辑变成了企业正在扩大生产的同时,伴随着堆积如山的未售出的商品,消费者支出正
在放缓,主次重心改变,后面的 with 伴随逻辑不对
E 项时态错,消费者支出已经放缓?逻辑错,with 结构不优。

856.

Thomas Mann’s novel Doctor Faustus offers an examination not only of how difficult it is to

reconcile reason, Will, and passion together in any art form, but also a skillfully navigated

exploration of the major concerns of modernism .

A.an examination not only of how difficult it is to reconcile reason, will, and passion together

in any art form, but

免费加入GMAT备考群、机经群,可加微信:jmtccca
免费领取GMAT资料、了解GMAT课程,可加微信:jmtccca

B.an examination not only about the difficulty of reconciling reason, will, and passion in any

art form, and

C.not only an examination of how difficult it is to reconcile reason, will, and passion in any

art form, and

D.not only an examination about the difficulty with reconciling reason, will, and passion

together in any art form, but

E.not only an examination of the difficulty of reconciling reason, will, and passion in any art

form, but

平行前后要求概念对等,结构对等,平行无歧义。
A 项 not only of 和 but also a skillfully navigated 不平行
B 项 not only about 和 but also a skillfully navigated 不平行
C 项 not only an examination of 和 and 不平行
D 项 not only an examination about 和 and 不平行
E 项 not only an examination of the difficulty of 和 but also a skillfully navigated exploration of the major
concerns of

857.
Upon their first encountering leaf-cutting ants in South America, the insects seemed
to some Europeans to be carrying bits of greenery to shade themselves from the
tropical sun—hence the sobriquet “parasol ants.”

A. Upon their first encountering leaf-cutting ants in South America, the insects
seemed to some Europeans to be-----------their 中 指 代 有 误 , Insects 不 可 能 去
encountering 逻辑不对。Upon 这个词:在…上面; 当…时候。
B. Upon their first encountering leaf-cutting ants in South America, some Europeans
thought they were-----------their 指谁?They 指谁?
C. On first encountering leaf-cutting ants in South America, it seemed to some
Europeans that the insects were------------it 指谁?
D. On first encountering leaf-cutting ants in South America, some Europeans thought
the insects were 在南美洲第一次遇到割叶蚂蚁时,一些欧洲人认为这些昆虫携带
着一些绿色植物来遮挡自己,而不是热带太阳——这就是绰号“阳伞蚂蚁”。
E. On their first encounter with leaf-cutting ants in South America, some Europeans
thought it was because the insects were-----------their 无指代,it 指 ants 也不对,
though 后面句式残缺且累赘

858.

免费加入GMAT备考群、机经群,可加微信:jmtccca
免费领取GMAT资料、了解GMAT课程,可加微信:jmtccca

By skimming along the top of the atmosphere, a proposed new style of aircraft could
fly between most points on Earth in under two hours, according to its proponents.

A. By skimming along the top of the atmosphere, a proposed new style of aircraft
could fly between most points on Earth in under two hours, according to its
proponents.据其支持者称,通过掠过大气层顶部,一种新型飞机可以在地球上大部分地点之间飞行不到两小时。注
意这里的主语应该堪称 a proposed new style of aircraft 作为一个整体, 这个飞机是可以被
by skimming 所修饰的。
B. By skimming along the top of the atmosphere, proponents of a proposed new style
of aircraft say it could fly between most points on Earth in under two hours.-----支持
者不能去掠过啊大气层,it 无指代
C. A proposed new style of aircraft could fly between most points on Earth in under
two hours, according to its proponents, with it skimming along the top of the
atmosphere.----its 无指代,it 无指代,according to its proponents 放句中无意义。
后面 with 结构做伴随状语逻辑不对
D. A proposed new style of aircraft, say its proponents, could fly between most
points on Earth in under two hours because of its skimming along the top of the
atmosphere.---------逗号不链接俩动词,俩 its
E. According to its proponents, skimming along the top of the atmosphere makes it
possible that a proposed new style of aircraft could fly between most points on Earth
in under two hours.----动名词作主语,掠过大气层让它变得可能,主语变逻辑变

859.

According to a recent study, retirees in the United States are four times more likely to give

regular financial aid to their children as to receive it from them.

A.retirees in the United States are four times more likely to give regular financial aid to their

children as

B.retirees in the United States are four times as likely to give regular financial aid to their

children as it is for them

C.retirees in the United States are four times more likely to give regular financial aid to their

children than

免费加入GMAT备考群、机经群,可加微信:jmtccca
免费领取GMAT资料、了解GMAT课程,可加微信:jmtccca

D.it is four times more likely for retirees in the United States to give regular financial aid to

their children than they are

E.it is four times as likely that retirees in the United States will give their children regular

financial aid as they are

主干:retirees are four times more likely to give aid to their children as to receive it from them,
逻辑为:根据最近的一项研究,美国的退休人员向他们的孩子提供定期的经济援助的可能
性是他们从孩子那里索取经济资助的四倍。
A 项逻辑没错,但 as 使用错误。。搭配 more than..
B 项 as it is for them 错误,it 无指代,多余,且后面未划线部分也有一个 it。
C 项正确,more than,且 than 前后 to 都平行。
D 项 than 后 they are 多余,比较对象不可比。
E 项错同 D, it is four times as likely that 很啰嗦,且 as 后是 are,前是 will give,不平行。
逻辑错:退休人将会给他们的孩子经济资助就像他们现在从孩子那里索取资助一样的很有
可能是 4 倍。

860.

Discussion of greenhouse effects have usually had as a focus the possibility of Earth growing

warmer and to what extent it might, but climatologists have indicated all along that

precipitation, storminess, and temperature extremes are likely to have the greatest impact on

people.

A.Discussion of greenhouse effects have usually had as a focus the possibility of Earth

growing warmer and to what extent it might,

B.Discussion of greenhouse effects has usually had as its focus whether Earth would get

warmer and what the extent would be,

C.Discussion of greenhouse effects has usually focused on whether Earth would grow

warmer and to what extent,

免费加入GMAT备考群、机经群,可加微信:jmtccca
免费领取GMAT资料、了解GMAT课程,可加微信:jmtccca

D.The discussion of greenhouse effects have usually focused on the possibility of Earth

getting warmer and to what extent it might,

E.The discussion of greenhouse effects has usually focused on whether Earth would grow

warmer and the extent that is,

A,Discussion 和 have 主谓不一致,and to what 无平行对象,排除。

B,has usually had as its focus 两个谓语动词,错,what 结构不简洁。

C,正确。Extent 这个词没有必要再在后面加代词或者其他名词性的表达来体现程度这层含

义。

D,the possibility 和 to what extend...不平行。It 多余。

E,And the extent 无平行对象,that 修饰 extent 啰嗦,排除。

861.

In the seventh century B.C., the Roman alphabet was adapted from the Etruscan alphabet,

which in turn had been adapted in the previous century from a western Greek alphabet, which

itself had been adapted earlier in the same century from the Phoenician alphabet.

A.which itself had been adapted earlier

B.adapting itself earlier

C.itself being adapted earlier

D.having been earlier adapted itself

E.earlier itself having been adapted

正确答案 A。在公元前 7 世纪,罗马字母表改编自伊特鲁里亚字母,伊特鲁里亚字母又是

在上个世纪从希腊字母改编而来,希腊字母又在在同一时期的更早些时候从腓尼基字母改编

而来。

B,C,D,E 用的都是一个 Ving 的形式,做状语修饰前面句子的就近动作,表示一个伴随或者

免费加入GMAT备考群、机经群,可加微信:jmtccca
免费领取GMAT资料、了解GMAT课程,可加微信:jmtccca

结果,多余,就是一个 which 就近修饰即可。然后反身代词 its 在 SC 题目里较为啰嗦一般

不用。

862.

The foundation works to strengthen local and regional agricultural markets and cooperating

with governments, improving access for farmers for productive resources such as land and

credit.

A.cooperating with governments, improving access for farmers for

B.cooperates with governments to improve access for farmers to

C.cooperate with governments for improvements of access for farmers to

D.cooperate with governments and improve accessibility for farmers for their

E.in cooperation with governments to improve access for farmers for

A.cooperating with 无平行对象,排除。表目的对象 to 优于 for

B,正确,cooperates 和 works 平行,与主语主谓一致。

C,cooperate 应该用单数, for improvements of access for farmers to 表达冗长。

D,cooperate 应该用单数,improve 表目的在这变成和主句谓语并列,且后面表达冗杂了。

E,in cooperation with 无平行对象。

863.

A professor at the university has taken a sabbatical to research on James Baldwin's books that

Baldwin wrote in France while he was living there.

A.on James Baldwin's books that Baldwin wrote in France while he was living there

B.about the books James Baldwin wrote in France

C.into James Baldwin's books written while in France

D.on the books of Jam es Baldwin, written while he lived in France

免费加入GMAT备考群、机经群,可加微信:jmtccca
免费领取GMAT资料、了解GMAT课程,可加微信:jmtccca

E.the books James Baldwin wrote while he lived in France

A,表达啰嗦 there 多余,且 research 是及物动词,后面直接加宾语即可。

B,同样的,research 在这直接加宾语,while 后省略的应该是主句主语,补进去之后逻辑不

对。

C,同样的,research 在这直接加宾语,while 应该接完整句子。

D,同样的,research 在这直接加宾语,written 就近修饰的话,修饰前面的 JB,逻辑不对。

E,正确。这所大学的一位教授休假去研究詹姆斯鲍德温在法国居住期间写的一些书。

864.

When working with overseas clients, an understanding of cultural norms is at least as important as

grasping the pivotal business issues for the global manager.

A.When working with overseas clients, an understanding of cultural norms is at least as important

as grasping the pivotal business issues for the global manager.

B.When they work with overseas clients, understanding cultural norms is at least of equivalent

importance to grasping the pivotal business issues for the global manager.

C.For global managers working with overseas clients, understanding cultural norms is at least as

important as grasping the pivotal business issues.

D.For global managers working with overseas clients, an understanding of cultural norms is at

least as important to them as that they grasp the pivotal business issues.

E.Global managers working with overseas clients find an understanding of cultural norms to be

equally important as grasping the pivotal business issues.

免费加入GMAT备考群、机经群,可加微信:jmtccca
免费领取GMAT资料、了解GMAT课程,可加微信:jmtccca

A.主语 understanding 不能 working with overseas,逻辑不对。主句后面逻辑能理解,主要是前

面错。

B.they 指代 understanding 逻辑还是错,equivalent 冗杂。

C.正确。介词短语放句首总领句子开头,结构清晰,表意明确。

D.them 多余

E.重心变,变成 managers find understanding,to be 表达啰嗦。

865.

Often major economic shifts are so gradual as to be indistinguishable at first from ordinary

fluctuations in the financial markets.

A.so gradual as to be indistinguishable

B.so gradual they can be indistinguishable

C.so gradual that they are unable to be distinguished

D.gradual enough not to be distinguishable

E.gradual enough so that one cannot distinguish them

A.正确。So ..as to=so..that

B.they 无指代

C.they 无指代,that 后定语从句修饰不简洁

D.enough 放置位置错,逻辑错

E.enough 放置位置错,逻辑错,one、them 指代不明,且整体句式杂糅

866.
Dinosaur tracks show them walking with their feet directly under their bodies, like
mammals and birds, not extended out to the side in the manner of modern reptiles.

A. Dinosaur tracks show them walking with their feet directly under their bodies,
like----them 无指代,Dinosaur tracks 和 mammals 不可比

免费加入GMAT备考群、机经群,可加微信:jmtccca
免费领取GMAT资料、了解GMAT课程,可加微信:jmtccca

B. Dinosaur tracks show that they walked with their feet directly under their bodies,
as do-------they 指谁完全指代指前面的 dinosaur tracks 逻但由于辑不对, their 逻
辑上指恐龙,这句话没有单独出现过恐龙,主语也是以 Dinosaur tracks 的形式出现
的,所以不能人为脑补,表意还是不清晰。
C. Dinosaurs left tracks that showed them walking with their feet directly under their
bodies, like----them 无指代,这里比动词,不用李克
D. The tracks that dinosaurs left show that they walked with their feet directly under
their bodies, as do----they 完全指代。Their 指恐龙。恐龙留下的足迹表明它们是用脚直接在身体
下面行走的

E. In the tracks they left, dinosaurs are shown walking with their feet under their
bodies, like----are shown 被动语态,不优选噢,are shown 后直接跟 walking 不符
合表达习惯,做动名词也不对 不用 like,这里是比动词

867.
Although when a hagfish is threatened, it will secrete slime that is small in quantity, it
expands several hundred times as it absorbs seawater, forming a slime ball that can
coat the gills of predatory fish and either suffocate them or distress them enough to
make them flee.

A. Although when a hagfish is threatened, it will secrete slime 分 泌 粘 液 that is


small in quantity,------although 后句法结构混乱,it 无指代,关系不清楚
B. Although a small quantity of slime is secreted by the hagfish, when
threatened------粘液不能被威胁
C. Although, when threatened, a hagfish will secrete slime that is small in
quantity,----逻辑还是错,鱼不能膨胀数百倍(不能脑补河豚,字面意思,鱼就是
不能膨胀)
D. Although the slime secreted by a threatened hagfish is small in quantity,---------结
合为划线句子,是粘液回吸收海水膨胀数百倍
E. Although the hagfish secretes a small quantity of slime when threatened,-----不是
盲鳗鱼吸收海水膨胀数百倍

868.
Officials at the United States Mint believe that the Sacagawea dollar coin will be used
more as a substitute for four quarters rather than for the dollar bill because of its
weight, only 8.1 grams, which is far less than four quarters, which weigh 5.67 grams
each.

A. more as a substitute for four quarters rather than for the dollar bill because of its
weight, only 8.1 grams, which is far less than----more rather than 搭配错误,use as
固定搭配,because 后面跟名词名词短语逻辑前后不对,1 美元硬币将更多地被
用作 4 个 25 美分硬币的替代品,而不是 1 美元纸币因为他的重量。就完了,?
很突兀。

免费加入GMAT备考群、机经群,可加微信:jmtccca
免费领取GMAT资料、了解GMAT课程,可加微信:jmtccca

B. more as a substitute for four quarters than the dollar bill because it weighs only
8.1 grams, far lighter than-----use more as 搭配不对,lighter 形容亮度,重量不用
lighter,用抽象的 less 更好。
C. as a substitute for four quarters more than for the dollar bill because it weighs only
8.1 grams, far less than---正确
D. as a substitute for four quarters more than the dollar bill because its weight of only
8.1 grams is far lighter than it is for---后面的 than it is 多余
E. as a substitute more for four quarters rather than for the dollar bill because its
weight, only 8.1 grams, is far less than it is for---------more rather than 搭配错误,than
it is 多余

869.
In the United States, less than half as many multifamily housing units were produced
in the 1990s than in each of the previous two decades.

A. less than half as many multifamily housing units were produced in the 1990s
than=========as 和 than 不搭配,不可能是 less than than。units 可数,不用 less
B. less than half as many multifamily housing units had been produced in the 1990s
as----1990 一般过去时,过去的过去错,units 可数,不用 less

C. there were less than half as many multifamily housing units produced in the 1990s
than---there be 句型冗杂,as 和 than 不搭配,units 可数,不用 less

D. fewer than half as many multifamily housing units were produced in the 1990s as
上世纪 90 年代的多户住宅单位数量不到前 20 年的一半。
E. fewer than half as many multifamily housing units had been produced in the 1990s
than----------as 和 than 不搭配

870.
Educator Maria Montessori believed that students be allowed to choose from among a
number of different lessons designed for the encouragement of their development as
thinkers and creators with individual learning and thinking styles.

A. that students be allowed to choose from among a number of different lessons


designed for the encouragement of------ for the encouragement 逻辑不通,be allowed
to 不能作谓语,也不是修饰。不可能理解成学生被设计为了这鼓舞
B. that students be allowed to choose between a number of different lessons designed
to encourage----还是 be allowed 修饰的问题,
C. that students should be allowed to choose among a number of different lessons
designed to encourage 应该允许学生在许多不同的课程中进行选择,这些课程旨
在鼓励他们成为具有个人学习和思考风格的思考者和创造者。

免费加入GMAT备考群、机经群,可加微信:jmtccca
免费领取GMAT资料、了解GMAT课程,可加微信:jmtccca

D. in allowing students to choose from among a number of different lessons were


designed for encouraging----in allowing 会有歧义,in 后面当成句子看逻辑也不对,
E. in allowing students to choose between a number of different lessons designed for
the encouragement of---错误同 A

871.
Analysts and media executives predict the coming year to be no less challenging than the
previous one had been for the company's C.E.O.

A.the coming year to be no less challenging than the previous one had been

B.the coming year to be no less challenging compared to the previous one

C.that the coming year would be no less challenging compared to the previous one

D.that the coming year will be no less challenging than the previous one had been

E.that the coming year will be no less challenging than the previous one

A.宾语从句 that 不能省,to be 不简洁,had done 时态不对


B.宾语从句 that 不能省,to be 不简洁
C.would 时态错,compared to 不简洁前后不可比
D.过去完成时时态多余
E.正确。分析人士和媒体高管预计,对公司的首席执行官来说,未来一年的挑战
不会比前一年少

872.
Scientists say that, by bathing the skin cells in extracts of immune cells, that human
skin cells in a test tube are made to behave as if they were immune system cells.

A. that human skin cells in a test tube are made to behave as if they were---俩 that 重
复,are made to behave 不符合表达习惯,生硬。
B. that human skin cells were to behave in a test tube as if they were---俩 that 仍然多
余,were to 这个虚拟放在细胞那里逻辑错
C. human skin cells in a test tube were made to behave as if-----as if 接未划线部分
之后逻辑错,as if 可做连词。
D. they have made human skin cells in a test tube that were behaving as---时态不对
E. they have made human skin cells in a test tube behave as if they were 正确。虚拟
语气使用正确,搭配 made sb.do 也正确。科学家们说,通过将皮肤细胞浸泡在免
疫细胞的提取物中,他们使试管中的人体皮肤细胞表现得好像是免疫系统细胞。

873.
Researchers now regard interferon as not a single substance, but it is Rather a biological

免费加入GMAT备考群、机经群,可加微信:jmtccca
免费领取GMAT资料、了解GMAT课程,可加微信:jmtccca

family of complex molecules that play an important, though not entirely defined, role in the
immune system.

A.as not a single substance, but it is rather a biological family of complex molecules that
play

B.as not a single substance but as a biological family of complex molecules playing

C.not as a single substance but as a biological family of complex molecules that play

D.not to be a single substance but rather a biological family of complex molecules playing

E.not as a single substance but instead as being a biological family of complex molecules
that play

A.Not..but 平行结构,原句 it 指代不明,排除。

B,not a... but as...不平行,排除。

C,正确.研究人员现在认为干扰素不是单一物质,而是复杂分子的生物家族在免疫系统中起

着重要的作用,尽管这还不完全确定。

D,not to be a single substance but rather a biological family 不平行,排除,

E,not as a single substance but instead as being a biological family 不平行,排除。

874.
The remarkable similarity of Thule artifacts throughout a vast region can, in part, be
explained as a very rapid movement of people from one end of North America to the other.

A.The remarkable similarity of Thule artifacts throughout a vast region can, in part, be
explained as

B.Thule artifacts being remarkably similar throughout a vast region, one explanation is

C.That Thule artifacts are remarkably similar throughout a vast region is, in part, explainable
as

D.One explanation for the remarkable similarity of Thule artifacts throughout a vast region
is that there was

E.Throughout a vast region Thule artifacts are remarkably similar, with one explanation for
this being

免费加入GMAT备考群、机经群,可加微信:jmtccca
免费领取GMAT资料、了解GMAT课程,可加微信:jmtccca

A.首先,原句的相似性被解释为移动,逻辑不对,排除。
B, being remarkably similar 表达不对,且 one explanation is movement 做同位语修饰 region 不
对,本身 explanation is movement 逻辑不对。排除。
C,的结构完美,但主语从句做主语逻辑还是不对,也是说人们的相似性可以解释为移动,不
对。
D 虽然罗嗦但是表意清晰,结构明确。对于人们这种手工艺品的相似性的解释是这里存在
着一个北美人民的的移动。正确。
E.无主句,排除,且 this 不能单独出现,being 的表达也不符合习惯。排除。

875.
Regulators are likely to end what are, in effect, long-standing exemptions permitting pilots of
small turboprop aircraft at small carriers to fly as much as 20 percent more hours per month
than pilots at larger airlines fly, with the consequence that some carriers could be forced to hire
additional pilots.

A.as much as 20 percent more hours per month than pilots at larger airlines fly, with the
consequence that

B.as many as 20 percent more hours per month as pilots at larger airlines, and

C.more hours per month, as much as 20 percent, than pilots at larger airlines; consequently

D.as much as 20 percent more hours per month as larger airlines’ pilots, so

E.as many as 20 percent more hours per month than pilots at larger airlines do, and
consequently

A.比动词,缺少助动词,with 在这里是介词,修饰和逻辑都有问题,不是带有这么一个后果。
B.比动词,缺少助动词,and 并列关系逻辑错。可数用 much,many 错
C.than 前后不可比
D.缺少助动词
E.正确,积累 and consequently 这种表达也是对的。

876.
Self-compassion is made up of mindfulness, the ability to manage thoughts and emotions
without being carried away or repressing them, common humanity, or empathy with the
suffering of others, and self-kindness, a recognition of your own suffering and a commitment
to solving the problem.
A.away or repressing them, common humanity, or empathy with the suffering of others,

免费加入GMAT备考群、机经群,可加微信:jmtccca
免费领取GMAT资料、了解GMAT课程,可加微信:jmtccca

B.away, or repression of them, and common humanity, or empathy with the suffering of
others,

C.away, or repressing them, common humanity, empathy with the suffering of others;

D.away or repressing them; common humanity, an empathy with the suffering of others;

E.away or repress them; common humanity, to empathize with the suffering of others

A.or 前后不平行
B.or 和 and 前后也不平行,语义连不上。
C.or 前后还是不平行。
D.正确。分号平行了 3 个事物。
E.分号前后不平行,和未划线的 and 连起来前后不平行。

877.
According to the laws of this nation, individuals are minors until they reach the age of
eighteen, although this is less in some countries and more in others.

A.although this is less in some countries and more in others

B.but this age is lower in some countries; higher in others

C.although in some countries, it is lower and in others it is higher

D.although it is less than that in some countries and more than that in others

E.but the relevant age is lower in some countries and higher in others

A.this 太口语不符合表达习惯且指代不明,although 关系不对


B.this 不符合表达习惯,分号多余。
C.表意不清淅,还是没说请什么东西 lower 和 higher,although 没有 but 关系好,
D.还是代词指代不明。Although 关系不好,
E.正确。明确主语是 relevant age,比其他选项号,也能对应 but 前面句子的语义。

878.
Rather than ignore a company that seems about to fail, investment analysts should recognize
that its reorganization and recent uptick in revenue, combined with its dynamic new
leadership, indicate that the firm’s prospects must be taken seriously.

免费加入GMAT备考群、机经群,可加微信:jmtccca
免费领取GMAT资料、了解GMAT课程,可加微信:jmtccca

A.Rather than ignore a company that seems about to fail,

B.Rather than ignoring a company that is about to seemingly fail,

C.Instead of a company that is seemingly about to fail being ignored,

D.Instead of ignore a company that seems about to fail,

E.In place of ignoring a company’s imminent failure seemingly about to occur,

A.正确。Rather 后面的动词形式跟着主句动词形式走,主句动词是原形,rather than 放句首


后跟的动词也就是原形。Seem about to=be about to
B.rather than 后 ignoring 形式不正确。
C.instead of 不优选,语义过于强硬,不是取代。being 不简洁。
D.instead of 不优选
E.in place of 不简洁,seemingly 错。

879.
Between 14, 000 and 8, 000 B. c. the ice cap that covered northern Asia,Europe, and
America began to melt, uncovering vast new areas that were to be occupied by migrating
peoples moving northward

A.began to melt, uncovering vast new areas that were to be occupied

B.began melting, to uncover vast new areas to be occupied

C.began, by melting, to uncover vast new areas for occupation

D.began, after melting, uncovering vast new areas which are to be occupied

E.would begin to uncover, through melting, vast new areas for occupation

A.正确。and 后结构正确,svo,doing,后面是一个状语伴随没问题,that 修饰 areas 也没问题,


后面的虚拟语气正确,意为将要占领。
B.不是 to uncover 逻辑不对,to be done 不简洁
C.重心变成开始去 uncover 了,occupation 词性变逻辑在这里是变化了的。
D.仍然是重心变,直接开始 uncovering.逻辑不对,
E.would 时态不对,分号拆分句子这里另起一个句子,逻辑不对,occupation 逻辑变

880.

免费加入GMAT备考群、机经群,可加微信:jmtccca
免费领取GMAT资料、了解GMAT课程,可加微信:jmtccca

Because property values sometimes fluctuate in response to economic conditions


beyond the purchaser's control, an investment in a home may underperform when
compared to that of other widely available classes of investments.

A. an investment in a home may underperform when compared to that of other


widely available classes of investments----that of 多余,when 后面主句主语补进去
对象不对,逻辑不对。
B. an investment in a home may underperform 运作结果差,表现不好 compared
with other widely available classes of investments 由于房产价值有时会随着经济形
势的变化而波动,超出了购买者的控制范围,因此,与其他广泛存在的投资类别
相比,对房产的投资可能表现不佳
C. an investment in a home may underperform when comparing it with other widely
available classes of investments---不是主动 comparing,it 无指代
D. compared to that of other widely available classes of investments, an investment
in a home may underperform----compared to that of compared to that of 错 , 谁
compared?缺主句
E. in comparison with that of other widely available classes of investments, an
investment in a home may underperform------in comparison with that of other 无 对
比,和就近对比逻辑也不对。缺主句

OG2021-880
Because property values sometimes fluctuate in response to economic conditions
beyond the purchaser's control, an investment in a home may underperform when
compared to that of other widely available classes of investments.

A:an investment in a home may underperform when compared to that of other widely
available classes of investments----that of 多余
B:an investment in a home may underperform 运作结果差,表现不好 compared
with other widely available classes of investments 由于房产价值有时会随着经济形
势的变化而波动,超出了购买者的控制范围,因此,与其他广泛存在的投资类别
相比,对房产的投资可能表现不佳
C:an investment in a home may underperform when comparing it with other widely
available classes of investments---不是主动 comparing,it 无指代
D:compared to that of other widely available classes of investments, an investment in
a home may underperform----compared to that of compared to that of 错 , 谁
compared?缺主句
E : in comparison with that of other widely available classes of investments, an
investment in a home may underperform------in comparison with that of other 无 对
比,和就近对比吗?逻辑也不对。缺主句

OG2021-881

免费加入GMAT备考群、机经群,可加微信:jmtccca
免费领取GMAT资料、了解GMAT课程,可加微信:jmtccca

Bengal-born writer, philosopher, and educator Rabindranath Tagore had the greatest
admiration for Mohandas K. Gandhi the person and also as a politician, but Tagore
had been skeptical of Gandhi’s form of nationalism and his conservative opinions
about India’s cultural traditions.
(A) for Mohandas K. Gandhi the person and also as a politician, but Tagore had been
(B) for Mohandas K. Gandhi as a person and as a politician, but Tagore was also
(C) for Mohandas K. Gandhi not only as a person and as a politician, but Tagore was
also
(D) of Mohandas K. Gandhi as a person and as also a politician, but Tagore was
(E) of Mohandas K. Gandhi not only as a person and as a politician, but Tagore had
also been

A 选项 and 前后不平行
B 正确。as a person and as a politician 平行,且逻辑意思合理。翻译:孟加拉出
生的作家、哲学家、教育家泰戈尔对甘地本人和作为政治家的甘地最为钦佩,但
泰戈尔也对甘地的民族主义形式和他对印度文化传统的保守看法表示怀疑。
C 选项 not only...but also 语义错误,这里并不是不仅。。。而且的递进关系,而
是转折。翻译:孟加拉出生的作家、哲学家、教育家泰戈尔对甘地最赞赏的不仅
是他作为个人和政治家,而且泰戈尔也对甘地的民族主义形式和他对印度文化传
统的保守看法表示怀疑。
D admiration of 在这里表达不合理,应该接 for, 且 also 应该放在 but 后的主句里,
翻译参考 B 选项。
E admiration of 在这里表达不合理,应该接 for,not only...but also 语义错误,这里并
不是不仅。。。而且的递进关系,而是转折。翻译见 C。

OG2021-882
Traffic safety officials predict that drivers will be equally likely to exceed the
proposed speed limit as the current one.
(A) equally likely to exceed the proposed speed limit as
(B) equally likely to exceed the proposed speed limit as they are
(C) equally likely that they will exceed the proposed speed limit as
(D) as likely that they will exceed the proposed speed limit as
(E) as likely to exceed the proposed speed limit as they are

A equally likely …as 是错误的搭配结构。


B equally likely …as 是错误的搭配结构。
C equally likely …as 是错误的搭配结构。
D as likely that 不如 likely to do 简洁,且 they 指代模糊。
E 正确。翻译见原句翻译

OG2021-883

Written early in the French Revolution, Mary Wollstonecraft’s A Vindication of the


Rights of Man (1790) and A Vindication of the Rights of Woman (1792) attributed

免费加入GMAT备考群、机经群,可加微信:jmtccca
免费领取GMAT资料、了解GMAT课程,可加微信:jmtccca

Europe’s social and political ills to be the result of the dominance of aristocratic
values and patriarchal hereditary privilege.
Mary Wollstonecraft’s A Vindication of the Rights of Man (1790) and A
Vindication of the Rights of Woman (1792) attributed Europe’s social and political
ills to be the result of
Mary Wollstonecraft’s A Vindication of the Rights of Man (1790) and A
Vindication of the Rights of Woman (1792) attributed Europe’s social and political
ills to result from
(C) Mary Wollstonecraft’s A Vindication of the Rights of Man (1790) and A
Vindication of the Rights of Woman (1792) attributed Europe’s social and political
ills to
(D) in A Vindication of the Rights of Man (1790) and A Vindication of the Rights
of Woman (1792),Mary Wollstonecraft attributed Europe’s social and political ills to
have been the result of
(E) Mary Wollstonecraft, in A Vindication of the Rights of Man (1790) and A
Vindication of the Rights of Woman (1792), attributed Europe’s social and political
ills to

A 选项 attributed to be 表达错误,且和 the result of 语义重复,attribute...to 把...归


因于...
B 选项 attributed to result from 表达错误,且也存在语义重复。
C 正确。
D 选项 Written 放在句首,后面句子的主语应该是法案,而不是 MW 这个人
E attribute 的 逻辑主语不对,应该是 vindication attribute 而不是人 attribute

OG2021-884

Using study groups managed by the principal popular organizations and political
parties, the Swedish public was informed by the government about energy and nuclear
power.
(A) the Swedish public was informed by the government about energy and nuclear
power
(B) the government informed the Swedish public about energy and nuclear power
(C) energy and nuclear power information was given to the Swedish public by the
government
(D) information about energy and nuclear power was given to the Swedish public by
the government
(E) the public of Sweden was given energy and nuclear power information by the
government

Using study groups 放在句首,直接判断后面句子的主语和该短语的关系,只有 B


选项是正确的。正确逻辑翻译:翻译:瑞典政府利用主要受欢迎的组织和政党管
理的研究小组,向瑞典公众介绍了能源和核能

免费加入GMAT备考群、机经群,可加微信:jmtccca
免费领取GMAT资料、了解GMAT课程,可加微信:jmtccca

A &Ethe Swedish public 做主语错误,公众利用研究小组,被征服通知...逻辑予以


错误

C&Dinformation 做主语逻辑错误,不可能是信息利用研究小组

OG2021-885

The use of the bar code, or Universal Product Code, which was created in part to
enable supermarkets to process customers at a faster rate, has expanded beyond
supermarkets to other retail outlets and have become readily accepted despite some
initial opposition when it was first introduced in 1974.
(A) have become readily accepted despite some initial opposition when it was first
introduced in 1974
(B) has become readily accepted despite some initial opposition when they were
first introduced in 1974
(C) have become readily accepted despite some initial opposition when first
introduced in 1974
(D) has become readily accepted despite some initial opposition when the bar code
was first introduced in 1974
(E) bar codes have become readily accepted despite some initial opposition when it
was first introduced in 1974
解析:句子结构:The use of the bar code...has expanded and have become....
A&C 主谓不一致,排除
B 选项的 they 在这里指代 Universal Product Code,应该用单数代词 it
D 正确。翻译:条形码的使用(产品代码)扩大了超市以外的其他零售网点,并
且尽管在 1974 年条形码首次引进引起了一些反对,但后来还是被欣然接受了。
E 选项 it 在这里指代 bar codes,应该用复数代词,而不是单数。

OG2021-886

Normally a bone becomes fossilized through the action of groundwater, which


permeates the bone washes away its organic components, and replaces them with
minerals.
A. which permeates the bone, washes away its organic components, and replaces
them
B. which permeates the bone, washes away its organic components, and those are
replaced
C. which permeates the bone, washing away its organic components, to be replaced
D. permeating the bone, washing away its organic components, to be replaced
E. permeating the bone, washing away its organic components and replacing them

解析:选 A

免费加入GMAT备考群、机经群,可加微信:jmtccca
免费领取GMAT资料、了解GMAT课程,可加微信:jmtccca

A 选 项 which 指 代 前 面 的 the action of groundwater , 后 面 是


permeates, washes, replaces 三个动作的平行.通常情况下,一根骨头会通过地
下水的作用而变成化石,因为地下水渗透到骨头中,冲走了它的有机成分,并用
矿物质取代了它们。
B 选项 and 后面是 those are replaced,和前面的动词不平行,those 指代不明。
C 选项三个动作不平行,没有连词连接。
D,E 选项逗号+doing 是状语表伴随或结果,逻辑不对,且 3 个动作没连词连接。

OG2021-887

The Organization of Petroleum Exporting Countries (OPEC) had long been expected
to announce a reduction in output to bolster sagging oil prices, but officials of the
organization just recently announced that the group will pare daily production by 1.5
million barrels by the beginning of next year, but only if non-OPEC nations, including
Norway, Mexico, and Russia, were to trim output by a total of 500,000 barrels a day.
(A) year, but only if non-OPEC nations, including Norway, Mexico, and Russia,
were to trim output
(B) year, but only if the output of non-OPEC nations, which includes Norway,
Mexico, and Russia, is trimmed
(C) year only if the output of non-OPEC nations, including Norway, Mexico, and
Russia, would be trimmed
(D) year only if non-OPEC nations, which includes Norway, Mexico, and Russia,
were trimming output
(E) year only if non-OPEC nations, including Norway, Mexico, and Russia, trim
output
解析:未划线部分宾语从句中 announced that the group will pare 时态是一般将来
时,所以 if 从句应该是一般现在时,即主将从现。所以只有 E 选项时态正确。

OG2021-888
Even with the proposed budget cuts and new taxes and fees, the city's projected deficit
for the next budget year is getting worse: administration officials announced that they
believe the gap will be $3.7 billion, a billion dollars over what it was predicted just
two months ago.

A.over what it was predicted---表示超过,不用 over,what 结构不优选,it 无指代,


时态不对
B.over the prediction from---dollars 和 prediction 不可比
C.more than it was predicted==it 无指代,时态不对

D.more than they had predicted 政府官员宣布,他们认为赤字将达到 37 亿美元,


比他们两个月前预测的多出 10 亿美元。
E:more than they predicted it---时态不对

免费加入GMAT备考群、机经群,可加微信:jmtccca
免费领取GMAT资料、了解GMAT课程,可加微信:jmtccca

OG2021-889
Over the past ten years cultivated sunflowers have become a major commercial crop,
second only to soybeans as a source of vegetable oil.
A second only to soybeans as a source of vegetable oil.
B. second in importance to soybeans only as a source of vegetable oil
C. being second in importance only to soybeans as a source of vegetable oil
D. Which, as a source of vegetable oil, is only second to soybeans
E. as a source of vegetable oil only second to soybeans

考点:句法结构+修饰
难度:600-650
解析:选 A
主干 sunflowers have become a major commercial crop,second only to soybeans..
副词 only 修饰且强调 soybeans 的 second,逻辑为向日葵已经成为一种主要的商业
作物,是仅次于大豆的作为植物油的来源。
B 项错误,人为添加了 importance,没意义,only 修饰 as a source of 去了,重心
变。
C 项 being 后面加名词,形容词,介词短语做修饰不简洁要删除,importance 多
余。
D 项有歧义造成就近修饰 commercial crop,only 修饰 second,逻辑错。
E 项 only 修饰 second 错,仅第二,还有可能是第三第四,违背了原文强调的是
soybeans 之后,没在其他之后。

OG2021-890
There are several ways to build solid walls using just mud or clay, but the most
extensively used method has been the forming of bricks out of mud or clay, and,after
some preliminary air drying or sun drying, they are laid in the wall in mud mortar.
(A) the forming of bricks out of mud or clay, and, after some preliminary air drying
or sun drying,they are laid
(B) forming the mud or clay into bricks, and, after some preliminary air drying or
sun drying, to lay them
(C) having bricks formed from mud or clay, and,after some preliminary air drying
or sun drying,they were laid
(D) to form the mud or clay into bricks, and, after some preliminary air drying or
sun drying, to lay them
(E) that bricks were formed from mud or clay, which,after some preliminary air
drying or sun drying,were laid

解析:A 选项 the most extensively used method the forming of bricks out of mud or
clay, and they are laid 两个主句放在一起逻辑不合理,and 后的主语 they 在结构上
应该指代 method,错误
B 选项 forming of bricks and to lay 不平行,错误
C 选项错误同 A

免费加入GMAT备考群、机经群,可加微信:jmtccca
免费领取GMAT资料、了解GMAT课程,可加微信:jmtccca

D 正确。翻译:有几种方法可以只用泥土或黏土来建造坚固的墙壁,但最广泛使
用的方法是将泥土或泥土变成砖块,经过初步的风干或太阳晒干后,把它们放在
泥灰浆中
E 选项 that...and which...引导的两个从句并列,改变了原句合理的的逻辑语义

OG2021-891
Scientists claim that the discovery of the first authenticated mammal bones in amber
could provide important clues of determining, in addition to how, when mammals
colonized the islands of the West Indies.

A.of determining, in addition to how, when mammals colonized the islands of the
West Indies-----of 不对,决定啥啥的线索
B.in the determination of how and when the islands of the West Indies were colonized
by mammals----决心的线索,and 不链接一个主一个从,逻辑还是不对
C.to determine how mammals colonized the islands of the West Indies and when they
did----表目的对,但是 and 前后还是逻辑和结构都不对
D.for determining when the islands of the West Indies were colonized by mammals
and how they were-----how 后面语义不完整
E.for determining how and when mammals colonized the islands of the West Indies
以确定哺乳动物如何和何时在西印度群岛的岛屿上定居

OG2021-892

In the 1940s popular magazines in the United States began to report on the private
lives of persons from the entertainment industry, In despite of the fact that they
previously had featured individuals in business and politics.
A. in despite of the fact that they previously had featured individuals
B. in spite of the fact previously that these publications featured articles on those
C. whereas previously there were those individuals featured in articles
D. whereas previously they individuals they featured were
E. whereas previously these publications had featured articles on individuals
考点:句法结构+修饰
难度:600-650
解析: 选 E
A.主干逻辑是 magazine 开始报道娱乐圈人的私生活,尽管他们之前刻画过政治
商界的个人。In despite of 短语搭配错误,只有 despite 或者 in spite of ,没有 in
despite of 的用法。had featured 正确,过去的过去刻画的人物,发生在 report 之
前。代词 They 前的复数名词不止一个,容易造成歧义。
B 项 previously 位置错误,those 想指人,但不明确,不轻易选有多个代词。
C 项 whereas 连词表然而没问题,there be 逻辑变,变成了过去报道私生活,然
而现在存在这样的个体,和原逻辑:之前报道私生活,然而之前还报道过政商的
个人。逻辑不对。those 无指代,featured 时态错误。

免费加入GMAT备考群、机经群,可加微信:jmtccca
免费领取GMAT资料、了解GMAT课程,可加微信:jmtccca

D 项对象不对等,they 代词有风险,前面是 magazines 报道了私生活,后面又说


然而 those individuals 怎么怎么样。
E 项 magazines 和 publications 对应,前后关系对等 featured 时态正确

OG2021-893

. In the early part of the twentieth century, many vacationers found that driving
automobiles and sleeping in tents allowed them to enjoy nature close at hand and tour
at their own pace, with none of the restrictions of passenger trains and railroad
timetables or with the formalities, expenses, and impersonality of hotels.
(A) with none of the restrictions of passenger trains and railroad timetables or with the
(B) with none of the restrictions of passenger trains, railroad timetables,nor
(C) without the restrictions of passenger trains and railroad timetables nor
(D) without the restrictions of passenger trains and railroad timetables or with the
(E) without the restrictions of passenger trains and railroad timetables or the

解析:A 选项 with none of...or with...逻辑错误,翻译:在二十世纪初,许多游客发


现,驾驶汽车和睡在帐篷里允许他们按照自己的节奏享受近距离的大自然,没有限
制的客运列车和铁路时间表,有手续,费用,和客观的酒店。
B 选项 with none...nor 搭配错误
C 选项 without...nor 搭配错误
D 选项 without...or with 逻辑表达 confusing
E 正确。翻译:在二十世纪初,许多游客发现,驾驶汽车和睡在帐篷里允许他们按
照自己的节奏享受近距离的大自然,没有限制的客运列车和铁路时间表,或手续,
费用,和客观的酒店。

OG2021-894

Over the next few years, increasing demands on the Chattahoochee River, which
flows into the Apalachicola River, could alter the saline content of Apalachicola Bay,
which would rob the oysters there of their flavor, and to make them decrease in size,
less distinctive, and less in demand.
(A) which would rob the oysters there of their flavor, and to make them decrease in
size,
(B) and it would rob the oysters there of their flavor, make them smaller,
(C) and rob the oysters there of their flavor, making them decrease in size,
(D) robbing the oysters there of their flavor and making them smaller,
(E) robbing the oysters there of their flavor, and making them decrease in size,

解析:A 选项,which 不能指代前面的整句话,修饰就近名词逻辑错误,且 and to


make them 前面没有平行的结构。翻译:在接下来的几年里,对流入阿巴拉契科
拉河的查特胡奇河的需求不断增加,可能会改变阿巴拉契科拉湾的盐含量,这将
会使牡蛎的味道受到破坏,并使它们的体积变小,不那么独特,需求也更少
B 选项 it 指代不明确,且 would...,make...句法结构错误,无连词连接。

免费加入GMAT备考群、机经群,可加微信:jmtccca
免费领取GMAT资料、了解GMAT课程,可加微信:jmtccca

C 选项 decrease in size,less distinctive, and less in demand 不平行


D 正确。翻译:在接下来的几年里,对流入阿巴拉契科拉河的查特胡奇河的需求
不断增加,可能会改变阿巴拉契科拉湾的盐含量,这将会使牡蛎的味道受到破坏,
并使它们的体积变小,不那么独特,需求也更少
E 选项 decrease in size,less distinctive, and less in demand 不平行

OG2021-895

Elizabeth Barber, the author of both Prehistoric Textiles, a comprehensive work on


cloth in the early cultures of the Mediterranean, and also of Women’s Work, a more
general account of early cloth manufacture, is an expert authority on textiles in
ancient societies.
(A) also of Women’s Work, a more general account of early cloth manufacture, is an
expert authority on
(B) also M/omen’s Work, a more general account of cloth manufacture, is an expert
authority about
(C) of 1/Vomen's Work, a more general account about early cloth manufacture, is an
authority on
(D) of Women's Work, a more general account about early cloth manufacture, is an
expert authority about
(E) Women's Work, a more general account of early cloth manufacture, is an
authority on
解析:
A 选项 both...and also of 不平行,且 expert authority 语义重复
B 选项错误同 A
C&D both...and of 后面结构不平行
E 正确。翻译:Elizabeth Barber,Prehistoric Textiles 和 Women's Work 两本书的
作者,是古代纺织界的权威。

OG2021-896

716. Digging in sediments in northern China, evidence has been gathered by scientists
suggesting that complex life-forms emerged much earlier than they had previously
thought.
(A) evidence has been gathered by scientists suggesting that complex life-forms
emerged much earlier than they had
(B) evidence gathered by scientists suggests a much earlier emergence of complex
life-forms than had been
(C) scientists have gathered evidence suggesting that complex life-forms emerged
much earlier than
(D) scientists have gathered evidence that suggests a much earlier emergence of
complex life-forms than that which was
(E) scientists have gathered evidence which suggests a much earlier emergence of
complex life-forms than that

免费加入GMAT备考群、机经群,可加微信:jmtccca
免费领取GMAT资料、了解GMAT课程,可加微信:jmtccca

解析:A 选项主语 evidence 和前面的 Digging 短语构成主动关系,逻辑错误。翻


译:在中国北方的沉积物中挖掘的证据被科学家收集,该证据表明,复杂的生命
形式出现比他们之前认为的要早得多。
B 错误同 A
C 正确。翻译:在中国北方的沉积物中挖掘的科学家收集证据,该证据表明,复
杂的生命形式出现比他们之前认为的要早得多。
D than that which...表达错误,且 that...定语从句修饰的是 evidence,改变了原句
suggesting that 引导宾语从句的合理的逻辑语义。翻译:在中国北方的沉积物中
挖掘的科学家们收集到了复杂的生命形式比之前认为的要早得多的证据
E 错误同 D。

OG2021-897
Employing many different techniques throughout his career, Michelangelo produced
a great variety of art works, including paintings, for example, in the Sistine Chapel, to
sculpture, for example, the statue of David.
(A) including paintings, for example, in the Sistine Chapel, to sculpture, for example,
(B) including paintings, for example, in the Sistine Chapel, to sculpture, like
(C) including paintings, such as those in the Sistine Chapel, and sculpture, as
(D) ranging from paintings, such as those in the Sistine Chapel, to sculpture, such as
(E) ranging from paintings, such as in the Sistine Chapel, and sculpture, such as

解析:A 选项 including...to...表达错误
B 选项 including...to...表达错误,like 表示举例错误
C 选项 as 错误,应该是 such as
D 正确。翻译:米开朗基罗在他的职业生涯中运用了许多不同的技巧,制作了许
多不同的艺术作品,从画作,比如西斯廷教堂的画作,到雕塑,比如大卫的雕像。

OG2021-898
According to a recent study of consumer spending on prescription medications,
increases in the sales of the 50 drugs that were advertised most heavily accounts for
almost half of the $20.8 billion increase in drug spending last year, the remainder of
which came from sales of the 9,850 prescription medicines that companies did not
advertise or advertised very little.
(A) heavily accounts for almost half of the $20.8 billion increase in drug spending last
year, the remainder of which came
(B) heavily were what accounted for almost half of the $20.8 billion increase in drug
spending last year; the remainder of the increase coming
(C) heavily accounted for almost half of the $20.8 billion increase in drug spending
last year, the remainder of the increase coming
(D) heavily, accounting for almost half of the $20.8 billion increase in drug spending
last year, while the remainder of the increase came
(E) heavily, which accounted for almost half of the $20.8 billion increase in drug
spending last year,with the remainder of it coming

免费加入GMAT备考群、机经群,可加微信:jmtccca
免费领取GMAT资料、了解GMAT课程,可加微信:jmtccca

解析:
A 选项句子结构:increase...accounts for...主谓不一致。翻译:根据最近一项有关
消费者在处方药上支出的研究,在去年 208 亿美元的药品支出增长中,广告宣传
力度最大的 50 种药品的销售增长占了近一半,其余部分来自 9850 种处方药的销
售,而这些药品公司很少宣传。
B 选项 were what accounted for 结构不简洁。
C 正确。翻译:根据最近一项有关消费者在处方药上的支出的研究,在去年 208
亿美元的药品支出增长中,广告宣传力度最大的 50 种药品的销售增长占了近一
半,剩下的增长来自 9850 种处方药的销售。
D&E 缺少谓语动词。

OG2021-899

Technically, “quicksand " is the term for sand that is so saturated with water as to
acquire a liquid’s character.
A. that is so saturated with water as to acquire a liquid's character
B. that is so saturated with water that it acquires the character of a liquid
C. that is saturated with water enough to acquire liquid characteristics
D. saturated enough With water so as to acquire the character of a liquid
E. saturated with water so much as to acquire a liquid character
考点:句法结构+修饰+比较
难度:650-700
解析:
主干是流沙是一种沙子的术语,啥术语呢?后面解释内容,用 that 更明确。然后
一个 so ..that 以致于表达出沙子因为太饱满,而使它具有了一种液体的属性
A 不选,so as to用法不对,应该是 so that 搭配。
B 正确
C 项 enough 位置错误,应该修饰 saturated 的。,to do 表目的逻辑错,不是去获
得一种液体属性。
D 项 so as to 连在一起用表目的,后一般接系动词
E 项同理 so much as to 错误,排除 D,E。

OG2021-900

At the end of 2001, motion picture industry representatives said that there were about
a million copies of Hollywood movies available online and
expected piracy to increase with high-speed Internet connections that become more
widely available.
(A) online and expected piracy to increase with high-speed Internet connections that
become more widely available
(B) online and expect the increase of piracy with the wider availability of high-speed
Internet connections

免费加入GMAT备考群、机经群,可加微信:jmtccca
免费领取GMAT资料、了解GMAT课程,可加微信:jmtccca

(C) online, and they expect more piracy to increase with the wider availability of
high-speed Internet connections
(D) online, and that they expected the increase of piracy as high-speed Internet
connections would become more widely available
(E) online, and that they expected piracy to increase as high-speed Internet
connections became more widely available

解析:
A 选项 expected 前要加 that,不加 that 会使得逻辑造成歧义,与 said 平行了,但
根据逻辑语义可知,是两个双宾语从句的并列,said that...and that...翻译:2001
年底,电影行业的代表说,在网上有大约一百万部好莱坞电影的拷贝预计盗版会
随着高速互联网连接的普及而增加。
B&C expect 时态错误,应该为过去时。
D 选项 would become 时态使用不准确,没有任何提示用过去将来时,且 they
expected the increase of piracy 强调的逻辑语义不够准确。翻译 2001 年底,电影
行业代表说,在线上可以买到大约 100 万部好莱坞电影,他们预计随着高速互联
网连接的普及,盗版的增加。
E 正确。翻译:2001 年底,电影行业代表说,在线上可以买到大约 100 万部好
莱坞电影,他们预计随着高速互联网连接的普及,盗版将会增加。

OG2021-901
Making things even more difficult has been general market inactivity lately, if not
paralysis, which has provided little in the way of pricing guidance.
(A) has been general market inactivity lately, if not paralysis, which has provided
(B) there is general market inactivity, if not paralysis, lately it has provided
(C) general market inactivity, if not paralysis, has lately provided
(D) lately, general market inactivity, if not paralysis, has provided
(E) is that lately general market inactivity, if not paralysis, which provides

解析 : A 选项 making things even more difficult 作 为主 语 ,则 使得 从 句 if not


paralysis 逻辑不合理,且 which 就近修饰 paralysis,逻辑不合理。翻译:令事情
变得更加困难的是近来市场普遍不活跃(如果令事情变得更加困难不是瘫痪的
话),这对定价指导几乎没有帮助。
B 选项 there be 和 it has provided...两个句子之间没有连词。
C 选项 making things even more difficult 这个修饰和主语 general market inactivity
之间无逗号,句子结构表达不合理。
D 正确。翻译:让事情最近变得更加困难的是,总体市场的不活跃(如果不是瘫
痪的话)在定价指导方面几乎没有提供什么帮助

OG2021-902
Ryunosuke Akutagawa’s knowledge of the literatures of Europe, China, and that of
Japan were instrumental in his development as a writer, informing his literary style as
much as the content of his fiction.

免费加入GMAT备考群、机经群,可加微信:jmtccca
免费领取GMAT资料、了解GMAT课程,可加微信:jmtccca

(A) that of Japan were instrumental in his development as a writer, informing his
literary style as much as
(B) that of Japan was instrumental in his development as a writer, and it informed
both his literary style as well as
(C) Japan was instrumental in his development as a writer, informing both his literary
style and
(D) Japan was instrumental in his development as a writer, as it informed his literary
style as much as
(E) Japan were instrumental in his development as a writer, informing both his literary
style in addition to

解析:A 选线 that of 多余,且主语为 knowledge,谓语动词应该用单数,were 错



B 选项 that of 多余,且 both...as well as...搭配错误
C 正确。翻译:RA 对欧洲、中国和日本文学的了解对他作为一个作家的发展起
到了重要的作用,这既体现在了他的文学风格中,又体现在了他的小说内容中。
D 选项 as 引导的状语从句改变了原句话的逻辑语义,且 as 从句表达的逻辑语义
不明确,表原因 or 时间?翻译:RA 对欧洲、中国和日本文学的了解对他作为一
个作家的发展起到了很大的推动作用,因为/同时这影响了他的小说内容,也影
响了他的文学风格。
E 选项主谓不一致,错误

OG2021-903
Many stock traders in the United States have set out to become global investors,
convinced that limiting their investments to the U.S. stock market, even though it is
certainly home to the stocks of some of the world`s great corporations, restricted their
gains.

Aeven though it is certainly

Bwhich, while it is certainly

Cdespite that that market is certainly

Dwhich, though certainly

Ealthough, certainly as

思路:
A. 正确 逻辑通顺,主句-v-ed modifier that 从句中插入 even though 作为连词
两套主谓- restrickted their gains 从句收尾。 limiting their investments 导致了
restricted gain

免费加入GMAT备考群、机经群,可加微信:jmtccca
免费领取GMAT资料、了解GMAT课程,可加微信:jmtccca

B. 套用 which 作为从句 只能就近修饰 us stock markt,从句中的逻辑主语是 us


market,while 从句中 it 也指代 us market 没问题,但是从句末尾中的 restricted
their gains 的逻辑主语变成了 market, 逻辑不通
C. despite that that, 有 点 搞 笑
D. 跟 b 选项类似 which 修饰 market, market restricted gain 也不对,though 里
面 主 语 是 market 但 是 没 有 谓 语
E. 使用 although 的话 逻辑主语事 limiting their investment restricted gain 没问
题,certainly as 就不正确了 未画线部分应该是修饰 stock market 的

OG2021-904
According to scientists who monitored its path, an expanding cloud of energized
particles ejected from the Sun recently triggered a large storm in the magnetic field
that surrounds Earth, which brightened the Northern Lights and also possibly
knocking out a communications satellite.
(A) an expanding cloud of energized particles ejected from the Sun recently triggered
a large storm in the magnetic field that surrounds Earth, which brightened the
Northern Lights and also possibly knocking
(B) an expanding cloud of energized particles ejected from the Sun was what recently
triggered a large storm in the magnetic field that surrounds Earth, and it brightened
the Northern Lights and also possibly knocked
(C) an expanding cloud of energized particles ejected from the Sun recently triggered
a large storm in the magnetic field that surrounds Earth, brightening the Northern
Lights and possibly knocking
(D) a large storm in the magnetic field that surrounds Earth, recently triggered by an
expanding cloud of energized particles, brightened the Northern Lights and it possibly
knocked
(E) a large storm in the magnetic field surrounding Earth was recently triggered by an
expanding cloud of energized particles, brightening the Northern Lights and it
possibly knocked

解析:A 选项句子结构中 which frightened...and knocking...不平行,且 knocking


不能作谓语,结构错误
B 选项句子结构 an expanding cloud was what...结构不简洁,且 and it brightened and
knocked 和前面的句子并列,逻辑错误。翻译:据监测其路径的科学家们说,最
近在地球周围的磁场中引发了一场巨大的风暴,并使北极光明亮,并可能摧毁了
一颗通讯卫星。
C 正确。翻译:据监测其路径的科学家称,一团不断膨胀的从太阳喷射出来的带
电粒子云最近在围绕地球的磁场中引发了一场大风暴,点亮了北极光,可能还摧
毁了一颗通讯卫星。
D 选项 particles ejected from the Sun 信息丢失,且句子结构错误,a storm 后无谓
语动词
E 选项 particles ejected from the Sun 信息丢失,且 and it knocked 这里做主句逻辑
错误。翻译:根据监测其路径的科学家们的说法,最近,围绕地球磁场的一场大

免费加入GMAT备考群、机经群,可加微信:jmtccca
免费领取GMAT资料、了解GMAT课程,可加微信:jmtccca

风暴,是由不断膨胀的带电粒子云所引发的,照亮了北极光,并它可能摧毁了一
颗通讯卫星。

OG2021-905

Because many of Australia's marsupials, such as the koala, are cute and cuddly, as
well as being biologically different than North American marsupials they have
attracted a lot of attention after their discovery in the 1700s.
A. being biologically different than North American marsupials, they have attracted a
lot of attention after
B. being biologically different from North American marsupials, they attracted a lot
of attention since
C. biologically different than North American marsupials, they attracted a lot of
attention since
D. biologically different than North American marsupials, they have attracted a lot of
attention after
E. biologically different from North American marsupials, they have attracted a lot of
attention since
考点:句法结构+动词形式+修饰
难度:700
解析:主干:because 一个原因状语从句,后面没有主句。原句 Different than 搭
配不对,应该是 different from,being 修饰不简洁多余,after 错误,应用现在完
成时。
同理排除 B,D.

E 项 different from,since 1700s,正确。因为澳大利亚的许多有袋类动物,比如考


拉,都很可爱,而且在生物性上和北美的有袋动物也所不同,自从它们在 18 世
纪被发现以来,它们吸引了很多的关注。
OG2021-906

Having been named for the Hellenic nymph who birthed to the river god Asopus, the
satellite named Thebe, in the inner ring of sixty-seven moons that orbit Jupiter, was
first observed in 1979.

A.Having been named for the Hellenic nymph who birthed to the river god Asopus,
the satellite named Thebe, in the inner ring of 67 moons that orbit Jupiter, was first
observed in 1979.

B.First observed in 1979, the satellite Thebe, named for the Hellenic nymph who was
birthed to the river god Asopus, is in the inner ring of 67 moons that orbit Jupiter.

C.In the inner ring of 67 moons that orbit Jupiter, the satellite Thebe, first observed in
1979 and named for a Hellenic nymph who birthed to the river god Asopus.

免费加入GMAT备考群、机经群,可加微信:jmtccca
免费领取GMAT资料、了解GMAT课程,可加微信:jmtccca

D.The satellite Thebe, named for the Hellenic nymph who birthed to the river god
Asopus and first observed in 1979, is in the inner ring of 67 moons to orbit Jupiter.

E.Thebe, a satellite first observed in 1979 and which was named for the Hellenic
nymph who was birthed to the river god Asopus, is in the inner ring of 67 moons to
orbit Jupiter.

解析:B
A:被命名为 a mythological nymph(having been named for a mythological nymph
who cared for the infant Jupiter)在本选项中是伴随状语,“Ida 被发现(the asteroid
named Ida was discovered in 1884)”是主句。这是不正确的。依据图形背景原则,
“被命名为 a mythological nymph”不能是“Ida 先被发现”的背景。因为,在逻辑上,
应是 Ida 先被发现,然后才是被命名,两者不具有时间轴上谁大于谁的问题。
C:本选项没有谓语动词(discovered 和 named 在本选项中均是过去分词,身前
需要加 be 动词(系动词)才能成为谓语动词(被动语态))。
D:句末的 to do 不定式(to orbit the Sun between Mars and Jupiter)在本选项中是
asteroids 的后置定语。其考查了“不定式和从句的区别”。用之于本题,主句“Ida
是在小行星带中的”事件的发生对“小行星带环绕太阳”这个事件发生与否没有直
接影响(你不能说,ida 在小行星带后,触发了这些小行星带环绕太阳吧?这两
个事件均是客观事实,不存在谁触发谁)。因此,只能用 that orbit the Sun between
Mars and Jupiter 这个从句的形式。
E:to orbit 不能做 asteriods 的定语错误同(D)。另外,本选项中的 an asteroid
discovered in 1884 和 which was named for a mythological nymph who cared for the
infant Jupiter 不平行。这是因为,前者是同位语,后者是定语,两者的功能不同
(如果两者都是定语,则可以平行),所以不能用 and 连接

OG2021-907

Custodian fees and expenses, as described in the statement of operations, include


interest expense incurred by the fund on any cash overdrafts of its custodian account
during the period.

A.include interest expense incurred by the fund on any cash overdrafts of its custodian
account during the period

B.are to include interest expenses on any cash overdrafts of its custodian account the
fund incurred during the period

C.includes interest expense the fund incurred during the period on any cash overdrafts
of its custodian account

D.may include interest expense during the period that the fund was to incur on any
cash overdrafts of its custodian account

免费加入GMAT备考群、机经群,可加微信:jmtccca
免费领取GMAT资料、了解GMAT课程,可加微信:jmtccca

including interest expense on any cash overdrafts of its custodian account incurred by
the fund during the period

A:正确。主句的主语为 fees and expenses,谓语是 include,宾语为 interest expense,


之后 为修 饰 expense 的 后 置定 语, 还 原 为 从 句形 式是 “interest expense that is
incurred by the fund…”。其中 during the period 是修饰 incurred 的状语。

B:从句子合理意思来看,the fund incurred 放在 account 之后跳跃修饰了 interest


expense,虽然跳跃修饰很常见,但此选项的跳跃修饰是毫无依据的。因为,跳
跃修饰往往是一种无奈之举,如:

a law of China that requires people to drive carefully.

这个词组中,of China 和 that requires people to drive carefully 都是修饰 law 的定


语,那按照顺序一个个放在 law 的身后,注定会造成跳跃修饰。这种跳跃修饰发
生的原则往往是先短后长,即长的修饰语放在后面进行跳跃修饰,这样句子看上
去也更加清晰。

在此选项中,“the fund incurred”很明显短于“on any cash overdrafts of its custodian


account ”,因此这个跳跃修饰是毫无逻辑的。

此外,be to do 表达的是将来时,但整句话中并没有表明这件事要在将来发生的
时间标志,因此用“are to include”是不合理的。

C:谓语动词采用了单数形式,造成了主谓不一致的错误。

D:during the period 放在 expense 之后,和 incur 隔了一个连词 that,造成 during


the period 错误的修饰了 expense,但合理的是“基金在这一期间承担费用”,即作
状语修饰动作,而不是“这一期间的利息费”,即作定语修饰名词。

E:including 为介词,作定语成分修饰名词,在这里它修饰了主语“保管费”,此
时,整句话缺少谓语动词,错误。

OG2021-908

Although some had accused Smith, the firm's network manager, of negligence when
the crucial data went missing, the CEO defused a situation that was quite tense with
her public statement that the debacle was not Smith's fault.
A、a situation that was quite tense with her public statement that the debacle was not
Smith's fault
B、a situation that was quite tense, by publicly stating that the debacle was not Smith's
fault

免费加入GMAT备考群、机经群,可加微信:jmtccca
免费领取GMAT资料、了解GMAT课程,可加微信:jmtccca

C 、 a situation, which was quite tense, by stating publicly that Smith was not
responsible for the debacle
D、a quite tense situation with a public statement about the debacle not being Smith's
fault
E、a quite tense situation by publicly stating the debacle not to have been Smith's fault
答案:B

A. 读完句子后意识到 这个主句的大意应该是 the CEO defused something by


doing something. 有一丝丝因果关系的意味 选项 a 中 用了 with 句子就变成了
CEO with something defused sth 因果的意味就没了
B. 正确, the CEO defused something,by doing sth。that 从句中也很简洁。稍微
强调了一下 CEO 的 action(publicly stating)
C. 迷惑选项, which 修饰 situation 没问题,语法上也是正确的,我有点不确
定 (publicly stating 和 stating publicly 的区别) that 从句后面有逻辑冲突。
debacle 是指崩坏(爱你 有道词典)原意应该是说 smith 弄丢了数据 但 崩坏不
是他的错, 该选项是 smith 不该为崩坏负责。 for example,我在街上丢了垃圾,
但是 街道的脏乱差不是我导致的。 (但是 如果说 我不该为此负责 就很渣男
了)
D. with 说 过 了 , about 。 。 not being 有 点 不 想 看

E. 前半部分 perfect defused sth by doing sth 但是 stating 后面 应是她 stating 或者
是 statement 的内容,而且 not to have been 有点新颖 时态也有待争议

OG2021-909

Many utilities obtain most of their electric power from large coal and nuclear
operations at costs that are sometimes two to three times higher as that of power from
smaller, more efficient plants that can both make use of waste heat and take advantage
of the current abundance of natural gas.
A two to three times higher as that of power from smaller, more efficient plants that
can both
B. higher by two to three times as that from smaller, more efficient plants that both
can
C. two to three times higher than those for power from smaller, more efficient plants
that can both
D. between two to three times higher as those for power from smaller, more efficient
plants that both can
E. between two to three times higher than from smaller, more efficient plants that they
can both.
考点:平行+比较
难度:700
解析:选 C

免费加入GMAT备考群、机经群,可加微信:jmtccca
免费领取GMAT资料、了解GMAT课程,可加微信:jmtccca

主干 utilities obtain power from coal and nuclear operationS at costs that..,意思是公
用事业获得的大部分电力来自于大型煤炭和核操作成本,这些成本又高出那些更
小更高效的利用余热和的天然气的工厂的 2-3 倍。
A 项:higher as 搭配错,that of无指代。
B 项 higher as 搭配错误。Both can ..and take 不平行
C 正确。Those 指 costs,make use of 和 take advantage of 平行。
D、E:表示倍数不存在 between to 这种用法,倍数要放比较级前面。E 项 than
后直接加 from 不对,比较的是 costs 和 power 了,但应该比较 costs。

OG2021-910
Five hundred million different species of living creatures have appeared on Earth,
nearly 99 percent of them vanishing.

A.Five hundred million different species of living creatures have appeared on Earth,
nearly 99 percent of them vanishing.-----不是主动消失
B .Nearly 99 percent of five hundred million different species of living creatures that
appeared on Earth have vanished.---(1)过去出现的 appeared 不可能现在还在消失。
时态尽量保持一致(2)E 更清晰一点,of 短语前置为了避免头重脚轻
C.Vanished are nearly 99 percent of the five hundred million different species of
living creatures that appeared on Earth.----还是形容词再聚首倒装
D.Of five hundred million different species of living creatures that have appeared on
Earth, nearly 99 percent of them have vanished.-----them 代词指代不清明确,画蛇添
足了
E.Of the five hundred million different species of living creatures that have appeared
on Earth, nearly 99 percent have vanished.---of 介词短语,真正主句是 99%已经销
声匿迹了,就是 species 消失,比 D 更准确。
补充:
语言表达现象:当主语的修饰语太长的时候要放在前面,避免头重脚轻。

OG2021-911
When viewed from the window of a speeding train, the speed with which nearby
objects move seems faster than that of more distant objects.

Athe speed with which nearby objects move seems faster than that of

Bthe speed that nearby objects move seems faster than for

Cthe speed of nearby objects seems faster than

Dnearby objects` speeds seem to be faster than those of

Enearby objects seem to move at a faster speed than do

免费加入GMAT备考群、机经群,可加微信:jmtccca
免费领取GMAT资料、了解GMAT课程,可加微信:jmtccca

考点:逻辑主语;比较
When viewed from the window…, …后面肯定需要出现实际可被 viewed 的物体,
speed 错,直接选 E
如果没看出逻辑主语,用比较也可以排出正确选项:
A. speed move faster than speed 的说法很奇怪——比较速度 move 的快慢?
B. 同 A;且前后比较对象不对等
C. 前后比较对象不对等——speed 与 objects 对比
D. 这个结构很别扭——nearby objects speeds 与 speeds of more distant objects 对
比 不好;seem to be 的用法在曼哈顿中属于 suspect,不如 seem to do;
E. 正确,恢复倒装就是 nearby objects seem to move at a faster speed than more
distant objects do

OG2021-912
Ramón pointed out that food high in whole-grain fiber creates the energy we need to
fight illnesses-as do vegetables and lean proteins.

Afiber creates the energy we need to fight illnesses-as do vegetables and lean proteins

Bfiber in addition to vegetables and lean proteins, create the energy we need to fight
illnesses

Cfiber creates the energy we need to fight illnesses, along with vegetables and lean
proteins

Dfiber, vegetables, and lean proteins creates the energy we need to fight illnesses

Efiber, as vegetables and lean proteins, create the energy we need to fight illnesses

A. 主语为 food, creates 主谓一致,正确。


B. 只有 and 并列可能的主语成分时,谓语动词才考虑复数形式,其他的“和”,“并
且“等 如: along with, in addition to, together with 还是根据主语判断单复数,
这里应该是 creates
C.fight 疾病,蔬菜和瘦蛋白?语义模糊
D.food high in fiber, (high in) vegetables, and (high in) lean proteins create 才对。
E.creates.

OG2021-913
The English physician Edward Jenner found that if experimental subjects were
deliberately infected with cowpox, which caused only a mild illness, they are immune
from smallpox.

A. which caused only a mild illness, they are immune from


B. causing only a mild illness, they become immune from

免费加入GMAT备考群、机经群,可加微信:jmtccca
免费领取GMAT资料、了解GMAT课程,可加微信:jmtccca

C. which causes only a mild illness, they are immune to


D. causing only a mild illness, they became immune from
E. which caused only a mild illness, they would become immune to

A.immune 固定搭配 后面要用介词 to


B.也涉及到固定搭配问题,同时 SVO,doing 这里的逻辑应该是后置定语,伴随
状语不能修饰 cowpox
C.Causes 和 become 时态都是不对的
D.错误前面也分析过了
E.正确,因为前面是 were infected with cowpox,they would 这里使用的是虚拟语

OG2021-914
As opposed to adults, pound for pound, children breathe twice as much air, drink two
and a half times as much water, eat three to four times as much food, and have more
skin surface area.

A.As opposed to 相反的 adults, pound for pound, children----和孩子什么相反没说


清楚。
B.Compared pound for pound with adults, children-----一磅一磅的和成年人比较,正
确,pound for pound 是个副词短语修饰 compare
C.Unlike an adult, pound for pound 均等的, children-----什么均等的还是没说清楚
D.Pound for pound, a child, unlike an adult, will---均等的放在前面无意义,什么均
等?
E.Pound for pound, children compared to adults will-----compared 和 will 不符合句式
结构,均等的,后面又说孩子要多呼吸多喝水,逻辑不对,这样就不均等了

OG2021-915
The final decades of the twentieth century not only saw an explosion of the literary
production among women, but there was also an intense Interest in the lives and
works Of women writers.
A. not only saw an explosion of the literary production among women, but there was
also
B. not only saw an explosion of literary production in women, but there was also
C. saw not only an explosion of literary production among women, but also
D. saw not only an explosion of the literary production by women, but it also saw
E. saw not only an explosion of literary production by women, but also saw
考点:句法结构+平行
难度:600-650
解析:C
A,B 选项 but 后面是个完整的句子,和 not only 前面的不平行

免费加入GMAT备考群、机经群,可加微信:jmtccca
免费领取GMAT资料、了解GMAT课程,可加微信:jmtccca

C 选项跟的都是名词 an explosion of 和 an intense Interest 平行。二十世纪的最后


几十年不仅见证了女性文学作品的爆炸式增长,同时也展现出了女性作家的生活
和作品产生了浓厚的兴趣。
D 选项 but 后面是个句子,和 not only 前面的不平行
E 选项 but 后面是谓语,和 not only 前面的不平行

OG2021-916
In the six-month period that ended on September 30, the average number of Sunday
papers sold by the company was 81,000 less than the comparable period a year ago.

A.the average number of Sunday papers sold by the company was 81,000 less than----
可数不用 less,sold 会给人谓语动词的歧义
B.on average, the number of Sunday papers sold by the company was 81,000 less than
it was----可数不用 less,同 A, 数字被 sold 歧义?
C.正确,the company sold an average of 81,000 fewer Sunday papers than in 与一年
前同期相比,该公司平均减少了 81000 份周日报纸的销量。
D.the company averaged sales of 81,000 fewer Sunday papers than what it did
in---company 后面缺谓语,前后没有可比性,what 结构罗嗦,it 指代不清
E.the average sale of Sunday papers for the company was 81,000 less than what they
were in----前后没有可比性,what 结构罗嗦,前后没有 were 这层含义

OG2021-917
Covering 71 percent of Earth`s surface, the oceans play an essential role in
maintaining the conditions for human existence on land, moderating temperature by
the absorption of heat and carbon dioxide, and giving pure water back to the
atmosphere through evaporation.
ACovering 71 percent of Earth`s surface, the oceans play an essential role in
maintaining the conditions for human existence on land, moderating
BCovering 71 percent of Earth`s surface and playing an essential role in maintaining
the conditions for human existence on land, the oceans moderate
CThe oceans cover 71 percent of Earth`s surface and play an essential role in
maintaining conditions for human existence on land, and by moderating
DThe oceans cover 71 percent of Earth`s surface, play an essential role in maintaining
the conditions for human existence on land, and moderate
EThe oceans cover 71 percent of Earth`s surface, playing an essential role in
maintaining the conditions for human existence on land, and they moderate

A 伴随状语,海洋(在保持人类生存的条件下)扮演了一个重要的角色,
moderating 和 giving pure water 是扮演重要角色的 2 个方面,(1)恒温(2)水
源,所以作伴随状语修饰 play 这个动作
B and giving pure water 无平行对象
C cover 和 play 在逻辑上不平行,因为起作用肯定是基于覆盖 71%的地球表面
也可以,假如不覆盖,再怎么样也不会对人类生存有影响/// and by moderating 前
面无平行

免费加入GMAT备考群、机经群,可加微信:jmtccca
免费领取GMAT资料、了解GMAT课程,可加微信:jmtccca

D 最后一个 and giving pure water 无平行


E 同D

OG2021-918
Michelangelo, it is believed, had made his sculpture of David using an eight-inch
plaster model that was recently discovered after being lost for nearly 300 years.

A.Michelangelo, it is believed, had made(过去完成时,已经做了) his sculpture of


David using an eight-inch plaster model that was recently discovered after
being--using 放在大卫后面修饰会哟歧义,had made 这个动词用的比较生硬,
B.An eight-inch plaster model is believed to have been used by Michelangelo for his
sculpture of David and recently discovered after it was----after 介词后不跟句子,被
动语态,句式糅杂
C.An eight-inch plaster model believed to have been used by Michelangelo for his
sculpture of David has been discovered after having been----一个八英寸的石膏模型
被认为是米开朗基罗用于他的雕塑大卫已经被发现后,失去了近 300 年。
D.It is believed that an eight-inch plaster model that Michelangelo used for his
sculpture of David and has recently been discovered after it was---缺谓语
E.It is believed that Michelangelo used an eight-inch plaster model for his sculpture of
David, and it was recently discovered after having been---两个 it,分别指代有问题

OG2021-919
Although the rise in the Producer Price Index was greater than expected, most
analysts agreed that the index was unlikely to continue going up and that inflation
remained essentially under control.
A.that the index was unlikely to continue going up and that inflation remained
B.that it was unlikely for the index continuing to go up and for inflation to remain
C.that the index was unlikely to continue to go up, with inflation to remain
D.on the unlikelihood that the index would continue going up and that inflation
remained
E.on the unlikelihood that the index would continue to go up and for inflation to
remain
看到 and 先看 parallelism, 两个 that 从句 彼此之间就句子的字面意思上没关系
(如果知道 ppi 的话 也不要去想之间的关系) 前后两部分 independent, 逻辑
通顺 正确
B. unlikely for the index continuing to go up… and 前后平行 但是意思有点不同,
原句意思是 index 不升了 和 inflation 还在掌控之中。 该选项,改变了意思。
变 成 index 不 升 了 而 且 inflation 也 不 在 掌 控 了 。
C. unlikely to continue to go up 有点小问题 不过无伤大雅,关键是 with 从句子
中来看,inflation 和 index 是没关系的 而且 我也不知道 with 是和 with analysts
还 是 the index 。
D E: analysts agreed on the unlikelihood 就跟表白的时候说:“我很喜欢你”和 “我

免费加入GMAT备考群、机经群,可加微信:jmtccca
免费领取GMAT资料、了解GMAT课程,可加微信:jmtccca

喜欢你的可能性很大”。磨磨唧唧。 d 中强行扯上 inflation 违背良心,e 中不平


行 and

OG2021-920
Just like the Internet today, often being called an “information superhighway,” the
telegraph was described in its day as an “instantaneous highway of thought.”
A:being,不可能是正在被称为。
B:being 去掉了,just like 也改成了 just as,可以留着。
C:as with 正如...一样可以接名词,但是 being 又错了
D:as is often the case with。。。情况常常和....一样。翻译就是(去掉插入语),
就像今日的互联网一样,电讯曾经被描述为“...”但是互联网是被描述为“信息高速
公路”的。互联网和电讯不能做到完全的统一。
E:similar to ...加名词,同位语歧义,可能修饰 today

OG2021-921
Severely hindered by problems with local suppliers, the fact that the AQ division also
had a new management team to adapt to was not seen by the board of directors as a
legitimate excuse for such low productivity.
A.Severely hindered by problems with local suppliers, the fact that the AQ division
also had a new management team to adapt to
B.Though severely hindered by local supply problems, the fact that the AQ division
also had a new management team to which to adapt
C.Severely hindered by problems with local suppliers, the AQ division also had to
adapt to a new management team, but this
D Severely hindered by local supply problems, that the AQ division also had to adapt
to a new management team
E Though severely hindered by problems with local suppliers, the AQ division`s also
having a new management team to which it had to adapt
A:the fact 主语和 hindered 之间不是被动的关系,逻辑主语错误
B:though hindered …状语从句省略,省略的主语和主句的主语一样,所以 the fact
也是和 hindered 构成被动语义,这个逻辑主语的意思不对
D:hindered 的逻辑主语是 that….主语从句,逻辑主语的意思不对
E:though hindered…状语从句省略,省略的主语和主句的主语一样 ,但是 the
AQ\\'s division\\' having…不能和 hindered 构成被动语义

OG2021-922

The Eastern State Penitentiary was established in 1822 by reformers advocating that
prisoners be held in solitary confinement and hard labor so as to reform them.
A. advocating that prisoners be held in solitary confinement and hard labor so as to
reform them

免费加入GMAT备考群、机经群,可加微信:jmtccca
免费领取GMAT资料、了解GMAT课程,可加微信:jmtccca

B. who were advocating prisoners to be held in solitary confinement and hard labor
for their reform
C. advocating solitary confinement and hard labor as the means to reform prisoners
D. who advocated solitary confinement and hard labor for the means of prisoner
reform
E. advocating as the means for prisoner reform solitary confinement and hard labor
考点:句法结构+修饰
难度:600-650
解析:选 C
主干是 The ESP was established .. by reformers advocating that prisoners be held in
solitary confinement and hard labor so as to reform them.逻辑是监狱在 1822 年由改
革者们建立,改革者主张把将囚犯单独监禁和劳役去改革他们。原文 advocating
that,后面强调的是主张的内容和既定事实,主张的是犯人被单独监禁和劳役为
了去改变他们,逻辑有误,这里应该就直接点名主张的两种方式,不用 that。So
as to 放一起表示为了,有 in order to 的含义
B 项定从 who 修饰人,逻辑重心变。
C 项正确,意为改革者倡导单独监禁,和用劳役作为改革他们的方式,逻辑清晰,
点名是 2 种方式。
D 项错误同 B,for 表为了错。
E 项逻辑错,主张为了单独监禁和劳役作为一种方式?错

OG2021-923
Some anthropologists believe that the genetic homogeneity evident in the world`s
people is the result of a "population bottleneck" -at some time in the past our
ancestors suffered an event, greatly reducing their numbers and thus our genetic
variation.
A at some time in the past our ancestors suffered an event, greatly reducing their
numbers
Bthat at some time in the past our ancestors suffered an event that greatly reduced
their numbers
Cthat some time in the past our ancestors suffered an event so that their numbers were
greatly reduced,
D some time in the past our ancestors suffered an event from which their numbers
were greatly reduced
Esome time in the past, that our ancestors suffered an event so as to reduce their
numbers greatly,

解析:B
注意句子主干是 anthropologists believe that... that ...,其中分号后面造成 reducing
的是一件事件,不是祖先去减少人数。
A:suffered 和 reduced 不平行,是谁 reducing 也没有说清楚,主语 anthropologists
believe that,后面没有解释好相信的是啥。项的分词从句出现在这么靠后的地方,
到底是定语从句——修饰 the event, 即 the event reduced their numbers 呢? 还是

免费加入GMAT备考群、机经群,可加微信:jmtccca
免费领取GMAT资料、了解GMAT课程,可加微信:jmtccca

状语从句修饰主句主谓——修饰 our ancestors suffered, 即 (when) our ancestors


reduced their(ancestors') numbers, our ancestors suffered an event. 这叫状语从句省
略修辞,省略引导词和相同主语,因此 reducing 的发起者默认是 ancestors。不读
句意,也可以确定 A 选项存在修饰级别错误的嫌疑。不优选
C:and 前面的逗号和后面你句子意思无法并列。So that 不能构成结果状语,导
致 thus 后面没有谓语
D:还原 their numbers were greatly reduced from an event,导致 thus 后面没有谓语
E:逗号后面不接 that 引导的非限定从。

OG2021-924

Through experimenting designed to provide information that will ultimately prove


useful in the treatment of hereditary diseases, mice have received bone marrow
transplants that give them a new gene
A. Through experimenting designed to provide information that will ultimately prove
B. Through experiments designed to provide information ultimately proving
C. In experimentation designed to provide information that ultimately proves
D. In experimenting designed to provide information ultimately proving
E. In experiments designed to provide information that will ultimately prove
考点:句法结构+修饰
难度:600-650
解析:选 E
主干是介词短语+真正主句 mice have received bone marrow transplants,原文逻辑
为:通过实验去提供信息将最终证明..,老鼠已经接受了骨髓移植。原文逻辑有问
题,老鼠应该是在实验中接受骨髓移植的,而不是通过实验,老鼠已经接受骨髓
移植。排除 A,B.
B 项应该是这个信息将会证明 will prove,而不是 proving
C 项不用动名词 experimenting,用名词形式 experiments 表示实验即可。
D 项错误同 B
E 项正确,在试验中去提供信息将最终证明...老鼠已经接受了骨髓移植

920
Just like the Internet today, often being called an “information superhighway,” the telegraph
was described in its day as an “instantaneous highway of thought.”

A.Just like the Internet today, often being

B.Just as the Internet is today often

免费加入GMAT备考群、机经群,可加微信:jmtccca
免费领取GMAT资料、了解GMAT课程,可加微信:jmtccca

C.As with the Internet being today often

D.As is often the case today with the Internet,

E.Similar to the Internet today, often

解析:AC 选项:being 冗余。


B 选项:正确,两个对象两种情况的比较。
D 选项:As is often the case today with A,B... 这种结构表示的是 A 和 B 都有同
一种情况,而这里是 A-C 情况,B-D 情况,C 和 D 是相似的,所以要用 as 引导
比较状语从句和主句比较。
E 选项:错同 D。

921
Severely hindered by problems with local suppliers, the fact that the AQ division also had a
new management team to adapt to was not seen by the board of directors as a legitimate
excuse for such low productivity.

A.Severely hindered by problems with local suppliers, the fact that the AQ division also had
a new management team to adapt to

B.Though severely hindered by local supply problems, the fact that the AQ division also had
a new management team to which to adapt

C.Severely hindered by problems with local suppliers, the AQ division also had to adapt to a
new management team, but this

D.Severely hindered by local supply problems, that the AQ division also had to adapt to a
new management team

E.Though severely hindered by problems with local suppliers, the AQ division’s also having
a new management team to which it had to adapt

解析:语义: 由于受到当地供应商问题的严重阻碍,AQ 部门也不得不适应新


的管理团队,但董事会并不认为这是生产率如此低下的正当理由。
句首出现分词结构,后面句子首先要给出的是句子的主语,然后考虑逻辑主语的
合理性。故排除 DE 选项,AB 选项 hindered 的逻辑主语错误,不是 fact,而是
AQ division。

免费加入GMAT备考群、机经群,可加微信:jmtccca
免费领取GMAT资料、了解GMAT课程,可加微信:jmtccca

922
The Eastern State Penitentiary was established in 1822 by reformers advocating that prisoners
be held in solitary confinement and hard labor so as to reform them .

A.advocating that prisoners be held in solitary confinement and hard labor so as to reform
them

B.who were advocating prisoners to be held in solitary confinement and hard labor for their
reform

C.advocating solitary confinement and hard labor as the means to reform prisoners

D.who advocated solitary confinement and hard labor for the means of prisoner reform

E.advocating as the means for prisoner reform solitary confinement and hard labor

解析:语义:ESP 监狱被改革者们建立,改革者主张单独监禁和劳役目的是为了
改造罪犯。
AB 选项,be held in solitary confinement and hard labor ,语义上不平行。
D 选项,for the means...介词搭配错误,for 表示为了,对于,这里应使用 as,作
为。E 选项, as the means for,介词搭配错误,跟方式,方法有关的,要用介词
to,再例:access to。

923
Some anthropologists believe that the genetic homogeneity evident in the world’s people is
the result of a “population bottleneck”—at some time in the past our ancestors suffered an
event, greatly reducing their numbers and thus our genetic variation.

A.at some time in the past our ancestors suffered an event, greatly reducing their numbers

B.that at some time in the past our ancestors suffered an event that greatly reduced their
numbers

C.that some time in the past our ancestors suffered an event so that their numbers were
greatly reduced,

D.some time in the past our ancestors suffered an event from which their numbers were
greatly reduced

E.some time in the past, that our ancestors suffered an event so as to reduce their numbers
greatly,

免费加入GMAT备考群、机经群,可加微信:jmtccca
免费领取GMAT资料、了解GMAT课程,可加微信:jmtccca

解析:CDE 选项首先 ancestor suffered an event,在过去的一段时间,at some time


时间状语,这里把 at 去掉,只剩下 some time 名词,则导致这句话双主语。再有
句子后面是 reduce A and B,那么 C 选项被动之后,后面未划线部分的 and 无法
平行。A 选项,reducing 的逻辑主语是 ancestor,不合理,应该是 event 使他们祖
先的人口数量减少。B 选项,加 that 是说明,这是对前面 believe that...这个从句
的解释说明,genetic homogeneity 对应 reduce our genetic variation ,population
bottleneck 对应 reduce their numbers。

924
Through experimenting designed to provide information that will ultimately prove useful in
the treatment of hereditary diseases, mice have received bone marrow transplants that give
them a new gene

A.Through experimenting designed to provide information that will ultimately prove

B.Through experiments designed to provide information ultimately proving

C.In experimentation designed to provide information that ultimately proves

D.In experimenting designed to provide information ultimately proving

E.In experiments designed to provide information that will ultimately prove

解析:注意 through 和 in,mice 老鼠不可能通过实验...,是在...的实验中的老鼠,


排除 AB 选项,D 选项,experimenting 的逻辑主语是 mice,不合理,experiment 是
具体化的实验,这个行为,experimentation 是实验法,是抽象的,一种方法,故
E 选项正确。

925
Linking arrangements among secondary schools and the workplace never evolved in the
United States as they have in most other developed countries.

A.among secondary schools and the workplace never evolved in the United States as they
have

B.in the United States among secondary schools and the workplace never evolved as they
did

C.between secondary schools and the workplace never evolved in the United States as

D.in the United States between secondary schools and the workplace never evolved as they
have

免费加入GMAT备考群、机经群,可加微信:jmtccca
免费领取GMAT资料、了解GMAT课程,可加微信:jmtccca

E.between secondary schools and the workplace never evolved in the United States as they
did

解析:A 选项,among 错误,两者间用 between;as 之前的动词是 evolved,as


之后应该用助动词的形式 did,如果 as 之前是 have evolved,as 之后才要用 have。
B 选项,among 错误,两者间用 between。C 选项,看上去正确,但省略的是 they
never did,表达的语义是中学和工作场所之间的安排在美国从未发生过变化,就
像大多数其他发达国家没有发生一样,但原句其实是要表达:在其他国家,这些
安排进展顺利,而在美国却没有。D 选项,助动词补错。E 选项,正确。

926
The ages of tropical rain forest trees provide critical information for understanding the
dynamics of tree populations, to determine historical patterns of disturbance, developing
sustainable forestry practices, and calculating carbon recycling rates.

A.to determine historical patterns of disturbance, developing sustainable forestry practices,


and calculating

B.to determine historical patterns of disturbance, develop sustainable forestry practices, and
to calculate

C.determining historical patterns of disturbance, developing sustainable forestry practices,


and calculating

D.determining historical patterns of disturbance, developing sustainable forestry practices,


and to calculate

E.determining historical patterns of disturbance, for developing sustainable forestry practices,


and for calculating

解析:语义:热带雨林树木的年龄提供了去理解树木种群动态,确定扰动的历史
模式,发展可持续森林运作方法,和计算碳回收率的重要信息。非常简单的平行
考点。从语义可以判断,是在考察 understanding,determining,developing,and
calculating 四个跟在 for 后面的动词的平行。
C 选 项 , 正 确 The ages of…provide critical information for
understanding.…determining…developing… and calculating…

927
The United Parcel Service plans to convert its more than 2,000 gasoline-powered trucks in the
Los Angeles area to run on cleaner-burning natural gas.

免费加入GMAT备考群、机经群,可加微信:jmtccca
免费领取GMAT资料、了解GMAT课程,可加微信:jmtccca

A.to convert its more than 2,000 gasoline-powered trucks in the LoS Angeles area to

B.to convert its more than 2,000 trucks in the Los Angeles area that are powered by gasoline
to.

C.on converting its more than 2,000 gasoline-powered trucks in the Los Angeles area that
will

D.for its more than 2,000 gasoline-powered trucks in the Los Angeles to convert to

E.that its more than 2,000 trucks in the Los Angeles area that are powered by gasoline will
convert to

解析:BCE 选项,that 定从跳跃修饰不优选;其次,plan to do 表示计划内容是


与动作 do 有关的,而 plan for sth,表示计划内容是与这个名词有关的,原句说
的是计划去 convert,所以 plan for sth/on sth 都不正确。A 选项,正确。

928
Foraging at all times of the day and night, but interspersing their feeding with periods of rest
that last between one and eight hours, a sperm whale could eat so much as a ton of squid a
day.

A.between one and eight hours, a sperm whale could eat so

B.between one and eight hours, sperm whales can eat as

C.between one to eight hours, sperm whales could eat as

D.from one to eight hours, sperm whales could eat so

E.from one to eight hours, a sperm whale can eat so

解析:between... and“在...和...之间”
From...to 更体现的是时间上的延续性
他晚上 8-10 点干活,就是 from to,中间没有暂停
如果说他在 8-10 点干活的期间可以休息,就是 between and
he can have a rest between 8 and 10,所以这里宜用 between... and
ADE 选项,so..as..的固定搭配错误;C 选项,between...to 固定搭配错误

929

免费加入GMAT备考群、机经群,可加微信:jmtccca
免费领取GMAT资料、了解GMAT课程,可加微信:jmtccca

In some types of pine tree,a thick layer of needles protects the buds from which new growth
proceeds; consequently they are able to withstand forest fires relatively well.

A.a thick layer of needles protects the buds from which new growth proceeds; consequently
they are able to withstand forest fires relatively well

B.a thick needle layer protects buds from where new growth proceeds, so that they can
withstand forest fires relatively well

C.a thick layer of needles protect the buds from which new growth proceeds; thus, they are
able to withstand relatively well any forest fires

D.since the buds from which new growth proceeds are protected by a thick needle layer,
consequently they can therefore withstand forest fires relatively well

E.because the buds where new growth happens are protected by a thick layer of needles,
they are able to withstand forest fires relatively easily as a result

解析:A 选项正确,主语 a thick layer


B 选项,from where 错误,前面没有地点可修饰
C 选项,主谓单复数不一致
D 选项,since,consequently,语义重复
E 选项,because,as a result,语义重复

930
The tourism commission has conducted surveys of hotels in the most popular resorts,with the
ultimate goal of reducing the guests who end up expressing overall dissatisfaction with the
service in the hotels.

A.with the ultimate goal of reducing the guests who end up expressing overall dissatisfaction
with the service in the hotels

B.with the goal to ultimately reduce the number of guests who end up expressing overall
dissatisfaction with the hotels’ service

C.ultimately with the goal to reduce expressions of overall dissatisfaction by the guests with
the hotel service

D.in an ultimate attempt to reduce the number of guests that ends up expressing overall
dissatisfaction with the hotels' service

E.with the ultimate goal of reducing the number of guests who express overall

免费加入GMAT备考群、机经群,可加微信:jmtccca
免费领取GMAT资料、了解GMAT课程,可加微信:jmtccca

dissatisfaction with the hotels' service

解析:语义:旅游事务署对最受欢迎的度假胜地的酒店进行了调查,最终目的是
减少对酒店服务整体不满的客人人数。
A 选项,语义不合理,减少的是不满的客人的数量,不是减少顾客
B 选项,未遵从原句合理的语义,原句表达的是最终目标(ultimate goal),B
选项表达的是最终减少(ultimately reduced)
CD 选项,修饰错误,错同 B
E 选项,正确

931
Unlike historical evidence of weather patterns in other regions of the world, which scientists
find abundantly represented in tree rings, ancient glacial ice, or layers of sediment from
seasonal plankton, the North Pole's clues about its past climates are almost nonexistent.

A.the North Pole's clues about its past climates are almost nonexistent

B.the North Pole does not offer many clues as to its past climates

C.clues to the past climates of the North Pole are almost nonexistent

D.there are few clues about past climates for the North Pole

E.the past climates of the North Pole do not offer many clues

解析:unlike 后的名词与主句主语比较,比较的是不同地区关于气候的线索
evidence 和 clues。A 选项:North Pole's clues 错误,比较的不是北极的线索,而
是关于气候的线索。
B 选项:主句主语是 the North Pole,比较对象错误。
C 选项:正确,evidence of weather patterns 和 clues to the past climates 比较对象
可比。
DE 选项:比较对象不可比。

932
A new study suggests that the conversational pace of everyday life may be so brisk it hampers
the ability of some children for distinguishing discrete sounds and words and, the result is, to
make sense of speech.
A.it hampers the ability of some children for distinguishing discrete sounds and words and,
the result is, to make

B.that it hampers the ability of some children to distinguish discrete sounds and words and,
as a result, to make

免费加入GMAT备考群、机经群,可加微信:jmtccca
免费领取GMAT资料、了解GMAT课程,可加微信:jmtccca

C.that it hampers the ability of some children to distinguish discrete sounds and words and,
the result of this, they are unable to make

D.that it hampers the ability of some children to distinguish discrete sounds and words, and
results in not making

E.as to hamper the ability of some children for distinguishing discrete sounds and words,
resulting in being unable to make

解析:so...that 如此...以至于,A 选项缺 that,排除;


B 选项,正确,hampers the ability (of some children ) to distinguish discrete sounds
and words and (, as a result,) to make sense of speech.
把中间括号的修饰拿掉,and 并列两个不定式;
C 选项,this 指代不清;
D 选项,语义重心在儿童...,it hampers..., and results in not making...平行,没有体
现出来强调的是儿童;
E 选项,ability for doing sth 固定搭配错误,应为 ability to do sth

933
The nineteenth-century chemist Humphry Davy presented the results of his early experiments
in his “Essay on Heat and Light,” a critique of all chemistry since Robert Boyle as well as a
vision of a new chemistry that Davy hoped to found.

A.a critique of all chemistry since Robert Boyle as well as a vision of a

B.a critique of all chemistry following Robert Boyle and also his envisioning of a

C.a critique of all chemistry after Robert Boyle and envisioning as well

D.critiquing all chemistry from Robert Boyle forward and also a vision of

E.critiquing all the chemistry done since Robert Boyle as well as his own envisioning of

解析:A 选项,正确。
vision 本身有名词的词性,则不选 envisioning 动名词充当名词,排除 BCE 选项。
D 选项,critiquing all chemistry from Robert Boyle forward and also a vision of,动
名词短语和名词形式不平行。

934
To attract the most talented workers, some companies are offering a wider range of benefits,
letting employees pick those most important to them.

免费加入GMAT备考群、机经群,可加微信:jmtccca
免费领取GMAT资料、了解GMAT课程,可加微信:jmtccca

A.benefits, letting employees pick those most important to them

B.benefits, letting employees pick the most important of them to themselves

C.benefits and letting employees pick the most important to themselves

D.benefits and let employees pick the most important to them

E.benefits and let employees pick those that are most important to themselves

解析:A 选项,正确;
B 选项,未遵从原句合理的逻辑语义,原句说的是 those most important,那些非
常重要的,也就是可选不止一个;而 B 选项是 the most important 选最重要的那
一个;
CDE 选项,句子结构错误,结构上 letting 修饰 offer,表示提供福利的方式,并
不是并列的另外一个动作,比如 wash and eat apple 是相对独立的动作互相不具有
修饰关系则使用并列结构。

935
Many of the earliest known images of Hindu deities in India date from the time of the
Kushan Empire, fashioned either from the spotted sandstone of Mathura or Gandharan grey
schist.

A.Empire, fashioned either from the spotted sandstone of Mathura or

B.Empire, fashioned from either the spotted sandstone of Mathura or from

C.Empire, either fashioned from the spotted sandstone of Mathura or

D.Empire and either fashioned from the spotted sandstone of Mathura or from

E.Empire and were fashioned either from the spotted sandstone of Mathura or from

解析:either ....or ....形式平行


排除 ABCD 选项,E 选项,正确

936
Tides typically range from three to six feet, but while some places show no tides at all, some
others, such as the Bay of Fundy, have tides of at least thirty feet and more.

A.some others, such as the Bay of Fundy, have tides of at least thirty feet and more

B.the others, such as the Bay of Fundy, that have tides of more than thirty feet

免费加入GMAT备考群、机经群,可加微信:jmtccca
免费领取GMAT资料、了解GMAT课程,可加微信:jmtccca

C.others, such as the Bay of Fundy, have tides of more than thirty feet

D.those at the Bay of Fundy, which has tides of more than thirty feet

E.the ones at the Bay of Fundy have tides of at least thirty feet and more

解析:A 选项:at least 和 more 重复


B 选项:句法结构错误,while SVO,S……缺谓语
DE 选项:those/ones 指代不清

937
A leading figure in the Scottish Enlightenment,Adam Smith’s two major books are to
democratic capitalism what Marx’s Das Kapital is to socialism.

A.Adam Smith’s two major books are to democratic capitalism what

B.Adam Smith’s two major books are to democratic capitalism like

C.Adam Smith’s two major books are to democratic capitalism just as

D.Adam Smith wrote two major books that are to democratic capitalism similar to

E.Adam Smith wrote two major books that are to democratic capitalism what

解析:考察同位语修饰,未划线部分是 a leading figure,所以后面的主语应为人,


即:Adam Smith,排除 ABC 选项;DE 选项,考察固定搭配:A is to B what C is
to D,意思是 A 之于 B,正如 C 之于 D。
语义:亚当·斯密写了两本重要的书它们对民主资本主义的意义就像马克思的《资
本论》对社会主义的意义一样

938
Researchers studying the brain scans of volunteers who pondered ethical dilemmas have
found that the basis for making tough moral judgments is emotion, not logic or analytical
reasoning.
A.the brain scans of volunteers who pondered ethical dilemmas have found that the basis for
making tough moral judgments is

B.the brain scans of volunteers who pondered ethical dilemmas and found the basis to make
tough moral decisions to be

免费加入GMAT备考群、机经群,可加微信:jmtccca
免费领取GMAT资料、了解GMAT课程,可加微信:jmtccca

C.the brain scans of volunteers pondering ethical dilemmas and found that the basis for
making tough moral decisions is

D.volunteers’ brain scans while pondering ethical dilemmas have found the basis to make
tough moral judgments to be

E.volunteers’ brain scans while they pondered ethical dilemmas have found that the basis for
making tough moral judgments is

解析:A 选项,正确
B 选 项 , Researchers studying the brain scans of volunteers who pondered ethical
dilemmas and found ,无谓语
C 选项,错同 B
D 选项,原句是说:研究人员对思考道德困境的志愿者进行了脑部扫描,D 选项,
while (researchers are) pondering 省略和主句相同的主语,逻辑语义错误
E 选项,they 指代不清

939
Rivaling the pyramids of Egypt or even the ancient cities of the Maya as an achievement, the
army of terra-cotta warriors created to protect Qin Shi Huang, China’s first emperor, in his
afterlife is more than 2,000 years old and took 700,000 artisans more than 36 years to
complete.

A.the army of terra-cotta warriors created to protect Qin Shi Huang, China’s first
emperor, in his afterlife is more than 2,000 years old and took 700,000 artisans more
than 36 years to complete

B.Qin Shi Huang, China’s first emperor, was protected in his afterlife by an army of
terra-cotta warriors that was created more than 2,000 years ago by 700,000 artisans
who took more than 36 years to complete it

C.it took 700,000 artisans more than 36 years to create an army of terra-cotta
warriors more than 2,000 years ago that would protect Qin Shi Huang, China’s first
emperor, in his afterlife

D.more than 2,000 years ago, 700,000 artisans worked more than 36 years to create
an army of terra-cotta warriors to protect Qin Shi Huang, China’s first emperor, in his
afterlife

E.more than 36 years were needed to complete the army of terra-cotta warriors that
700,000 artisans created 2,000 years ago to protect Qin Shi Huang, China’s first
emperor, in his afterlife

免费加入GMAT备考群、机经群,可加微信:jmtccca
免费领取GMAT资料、了解GMAT课程,可加微信:jmtccca

解析:分词短语置于句首,后面的句子要首先给出可以做前面分词结构的逻辑主
语的名词。故排除 BCDE 选项,A 选项,正确:“Rivaling the pyramids of Egypt or
even the ancient cities of the Maya as an achievement”的逻 辑主 语是“the army of
terra-cotta warriors” , 表 意为 “the army of terra-cotta warriors” 和“the pyramids of
Egypt”or“cities of the Maya”相媲美。

940
In California, a lack of genetic variation in the Argentine ant has allowed the species to spread
widely;due to their being so genetically similar to one another, the ants consider all their
fellows to be a close relative and thus do not engage in the kind of fierce intercolony struggles
that limits the spread of this species in its native Argentina.

A.due to their being so genetically similar to one another, the ants consider all their fellows
to be a close relative and thus do not engage in the kind of fierce intercolony struggles that
limits

B.due to its being so genetically similar, the ant considers all its fellows to be a close relative
and thus does not engage in the kind of fierce intercolony struggles that limit

C.because it is so genetically similar, the ant considers all its fellows to be close relatives
and thus does not engage in the kind of fierce intercolony struggles that limits

D.because they are so genetically similar to one another, the ants consider all their fellows to
be close relatives and thus do not engage in the kind of fierce intercolony struggles that limit

E.because of being so genetically similar to one another, the ants consider all their fellows to
be a close relative and thus do not engage in the kind of fierce intercolony struggles that limits

解析:AB 选项,due to 修饰名词/名词短语的原因,不作状语修饰句子


C 选项,it is so genetically similar,逻辑错误,两者才能 similar
E 选项,struggles that limits,单复数不一致

941
Next month, state wildlife officials are scheduled to take over the job of increasing the wolf
population in the federally designated recovery area, the number of which will
however ultimately be dictated by the number of prey in the area.

A.area, the number of which will however

B.area; the size of the population, however, will

免费加入GMAT备考群、机经群,可加微信:jmtccca
免费领取GMAT资料、了解GMAT课程,可加微信:jmtccca

C.area, however the number of wolves will

D.area; the number of which will, however,

E.area, when the size of the population will, however

解析:A 选项:the number of which 引导的是定语从句。定语和状语均须满足就


近修饰原则,因此本选项中该定语从句会修饰 designated recovery area。在逻辑
上,被该地区的猎物的数量决定的应该是狼的数量,而不是“地区”。
B 选项:正确。
C 选项:however 是副词而不是连词,因此,are scheduled 句和 will be dictated
句之间缺少连词。
D 选项:the number of which 引导的是定语从句,其修饰对象错误同选项(A)。另
外,分号一般用于连接个分句,不能连接一个主句和一个定语从句。
E 选项:连词 when 将 state wildlife officials are scheduled to take over the job of
increasing the wolf population in the federally designated recovery area 和 the size of
the population will ultimately be dictated by the number of prey in the area 塑造成了
时间(同时)关系。显然地,这两句话并非同一时间发生,也不能用后者来修饰
前者,两句分别讲了两个方面的事情,一个是接管工作,另外一个是狼的数量由
猎物数量决定。

942
About 5 million acres in the United States have been invaded by leafy spurge, a herbaceous
plant from Eurasia with milky sap that gives mouth sores to cattle, displacing grasses and
other cattle food and rendering rangeland worthless.

A.States have been invaded by leafy spurge, a herbaceous plant from Eurasia with milky sap
that gives mouth sores to cattle, displacing grasses and other cattle food and rendering

B.States have been invaded by leafy spurge, a herbaceous plant from Eurasia,with milky sap,
that gives mouth sores to cattle and displaces grasses and other cattle food, rendering

C.States have been invaded by leafy spurge, a herbaceous plant from Eurasia having milky
sap that gives mouth sores to cattle and displacing grasses and other cattle food, rendering

D.States, having been invaded by leafy spurge, a herbaceous plant from Eurasia with milky
sap that gives mouth sores to cattle, displaces grasses and other cattle food, and renders

E.States, having been invaded by leafy spurge, a herbaceous plant from Eurasia that has
milky sap giving mouth sores to cattle and displacing grasses and other cattle food rendering

解析:原句语义不合理,About 5 million acres in the United About 5 million acres in the


United States have been invaded by leafy spurge, a herbaceous plant from Eurasia with milky sap

免费加入GMAT备考群、机经群,可加微信:jmtccca
免费领取GMAT资料、了解GMAT课程,可加微信:jmtccca

that gives mouth sores to cattle, displacing grasses and other cattle food and rendering rangeland
worthless.
原句结构,displacing 和 rendering 作状语,就近修饰动词 gives,这里表结果还是
表伴随都不合理,milky sap 有害植物的白色汁液给牛嘴巴伤害,跟替换草地没
有修饰关系;合理的语义应该是这种有害的植物,给牛嘴巴伤害并且替换了草地
和其它牛可以吃的植物,这两种不好的情况 rendering 使得大草原变得没有价值,
即,rendering 作状语修饰 gives and displaces,表结果,B 选项,正确。C 选项,
rendering 就近修饰 having 和 displacing,语义不合理,草有白色汁液就使得大草
原没有价值?DE 选项,都是修饰成分,主句无谓语。

943
While it costs about the same to run nuclear plants as other types of power plants, it is the
fixed costs that stem from building nuclear plants that makes it more expensive for them to
generate electricity.

A.While it costs about the same to run nuclear plants as other types of power plants, it is the
fixed costs that stem from building nuclear plants that makes it more expensive for them to
generate electricity.

B.While the cost of running nuclear plants is about the same as for other types of power
plants, the fixed costs that stem from building nuclear plants make the electricity they
generate more expensive.

C.Even though it costs about the same to run nuclear plants as for other types of power
plants, it is the fixed costs that stem from building nuclear plants that makes the electricity
they generate more expensive.

D.It costs about the same to run nuclear plants as for other types of power plants, whereas
the electricity they generate is more expensive, stemming from the fixed costs of building
nuclear plants.

E.The cost of running nuclear plants is about the same as other types of power plants, but the
electricity they generate is made more expensive because of the fixed costs stemming from
building nuclear plants.

解析:语义:虽然运行核电站的成本与其他类型的核电站差不多,但建造核电站
的固定成本使它们产生的电力更加昂贵。
A 选项,固定成本 fixed costs 使它们产生的电力更加昂贵,makes 单复数错误。
B 选项,正确
C 选项,错同 A
D 选项,they 指代不清
E 选项,The cost of... is about the same as... power plants,比较对象不可比

免费加入GMAT备考群、机经群,可加微信:jmtccca
免费领取GMAT资料、了解GMAT课程,可加微信:jmtccca

944
The 32 species that make up the dolphin family are closely related to whales and in
fact include the animal known as the killer whale, which can grow to be 30 feet long and
is famous for its aggressive hunting pods.

A.include the animal known as the killer whale, which can grow to be 30 feet long and is

B.include the animal known as the killer whale, growing as big as 30 feet long and

C.include the animal known as the killer whale, growing up to 30 feet long and being

D.includes the animal known as the killer whale, which can grow as big as 30 feet long and
is

E.includes the animal known as the killer whale, which can grow to be 30 feet long and it is

解析:A 选项,正确
B 选项,As big as 30 feet long,表意不清,要么和...东西一样大,要么长到多长
C 选项,grow up to 错误,要么 grow up 长大,要么 grow to 长到...
DE 选项,32species 和 includes 主谓单复数不一致,排除

945
The first trenches that were cut into a 500-acre site at Tell Hamoukar, Syria, have yielded
strong evidence for centrally administered complex societies in northern regions of the
Middle East that were arising simultaneously with but independently of the more celebrated
city-states of southern Mesopotamia, in what is now southern Iraq.
A.that were cut into a 500-acre site at Tell Hamoukar, Syria, have yielded strong evidence
for centrally administered complex societies in northern regions of the Middle East that were
arising simultaneously with but

B.that were cut into a 500-acre site at Tell Hamoukar, Syria, yields strong evidence that
centrally administered complex societies in northern regions of the Middle East were arising
simultaneously with but also

C.having been cut into a 500-acre site at Tell Hamoukar, Syria, have yielded strong evidence
that centrally administered complex societies in northern regions of the Middle East were
arising simultaneously but

D.cut into a 500-acre site at Tell Hamoukar, Syria, yields strong evidence of centrally
administered complex societies in northern regions of the Middle East arising simultaneously
but also

免费加入GMAT备考群、机经群,可加微信:jmtccca
免费领取GMAT资料、了解GMAT课程,可加微信:jmtccca

E.cut into a 500-acre site at Tell Hamoukar, Syria, have yielded strong evidence that
centrally administered complex societies in northern regions of the Middle East arose
simultaneously with but

解析:后面讲的是 evidence 的内容,解释抽象名词的内容用 that 同位语从句,排


除 evidence for/of,即 AD 选项。
B 选项,主谓单复数不一致(trenches... yields)
C 选项,having done/having been done 不能做定语修饰名词
E 选项,正确

946
Companies are relying more and more on networked computers for such critical tasks as
inventory management, electronic funds transfer, and electronic data interchange, in which
standard business transactions are handled via computer rather than on paper.

A.in which standard business transactions are handled via computer rather than on paper

B.where computers handle standard business transactions rather than on paper

C.in which computers handle standard business transactions instead of on paper

D.where standard business transactions are handled, not with paper, but instead via computer

E.in which standard business transactions are being handled via computer, in place of on
paper

解析:A 选项:正确。
B 选项:本选项中,computers 主动发出 handle standard business transactions 这个
动作。在逻辑上,只能是人们“通过电脑”来处理商业交易,而不能电脑自己处理
商业交易,另外,where 做定语从句的引导词时,其先行词只能是人能到达的实
际地点,而本题中先行词是 critical tasks,所以应改为 in which,修饰抽象的地点。
C 选项:错同 B。
D 选项:where 应用于修饰地点,此处不正确。
E 选项:being 多余,这里是陈述理论,不需要正在的语义

947
Combining enormous physical strength with higher intelligence, the Neanderthals appear as
equipped for facing any obstacle the environment could put in their path, but their relatively
sudden disappearance during the Paleolithic era indicates that an inability to adapt to some
environmental change led to their extinction.

A.appear as equipped for facing any obstacle the environment could put in their path,

免费加入GMAT备考群、机经群,可加微信:jmtccca
免费领取GMAT资料、了解GMAT课程,可加微信:jmtccca

B.appear to have been equipped to face any obstacle the environment could put in their path,

C.appear as equipped to face any obstacle the environment could put in their paths,

D.appeared as equipped to face any obstacle the environment could put in their paths,

E. appeared to have been equipped for facing any obstacle the environment could put in their
path,

解析:appear as 后接名词,表示“以...身份出现” appear to 后面接动词,表示“看


起来像”,此题很明显是接动词,可排除 ACD 选项;根据 but 后的 indicates 主从
句时态一致,可判断整句话是一般现在的时态,而不是过去时态,排除 E;B 选
项,正确。

948
To map Earth’s interior, geologists use a network of seismometers to chart seismic waves that
originate in the earth’s crust and ricochet around its interior, most rapidly traveling through
cold, dense regions and slower through hotter rocks.

A.interior, most rapidly traveling through cold, dense regions and slower

B.interior, which travel most rapidly through cold, dense regions, and more slowly

C.interior, traveling most rapidly through cold, dense regions and more slowly

D.interior and most rapidly travel through cold, dense regions, and slower

E.interior and that travel most rapidly through cold, dense regions and slower

解析:A 选项:slower 不正确,slower 虽然既是 adj 也是 adv,但是这里与前面


的 rapidly 形式不平行,所以改成 slowly 更优。
B 选项:定从就近修饰 interior 不合理。
C 选项:正确,现在分词短语 traveling most rapidly through cold, dense regions 做
状语就近修饰 ricochet,弹跳,表示弹跳的方式,travel 的逻辑主语为 waves。
DE 选项:并列关系错误

949
Prices at the producer level are only 1.3 percent higher now than a year ago and are going
down, even though floods in the Midwest and drought in the South are hurting crops and
therefore raised corn and soybean prices.

A.than a year ago and are going down, even though floods in the Midwest and drought in the

免费加入GMAT备考群、机经群,可加微信:jmtccca
免费领取GMAT资料、了解GMAT课程,可加微信:jmtccca

South are hurting crops and therefore raised

B.than those of a year ago and are going down, even though floods in the Midwest and
drought in the South are hurting crops and therefore raising

C.than a year ago and are going down, despite floods in the Midwest and drought in the
South, and are hurting crops and therefore raising

D.as those of a year ago and are going down, even though floods in the Midwest and drought
in the South hurt crops and therefore raise

E.as they were a year ago and are going down, despite floods in the Midwest and drought in
the South, and are hurting crops and therefore raising

解析:A 选项,floods... are hurting crops and therefore raised corn and soybean prices.
raised 被动错误,洪水提升劳作物的价格,是主动关系,不是被动关系。
B 选项,正确,prices are higher now than those of a year ago 不同时间的价格比较。
C 选项,despite floods in the Midwest and drought in the South, and are hurting,句
子结构错误,are hurting 的主语应该是 floods。
DE 选项,比较级的比较,只能用 than,as 用于同级比较;

950
Fossils of the arm of a sloth found in Puerto Rico in 1991, and dated at 34 million years old,
made it the earliest known mammal of the Greater Antilles Islands.

A.sloth found in Puerto Rico in 1991, and dated at 34 million years old, made it the earliest
known mammal of

B.sloth, that they found in Puerto Rico in 1991, has been dated at 34 million years old, thus
making it the earliest mammal known on

C.sloth that was found in Puerto Rico in 1991, was dated at 34 million years old, making this
the earliest known mammal of

D.sloth, found in Puerto Rico in 1991, have been dated at 34 million years old, making the
sloth the earliest known mammal on

E.sloth which, found in Puerto Rico in 1991, was dated at 34 million years old, made the
sloth the earliest known mammal of

解析:A 选项,句子主干 fossils made it the earliest mammal,逻辑语义错误,应


该是化石在 34 million years 之前就被发现这整件事 made it the earliest mammal,
而 fossil。

免费加入GMAT备考群、机经群,可加微信:jmtccca
免费领取GMAT资料、了解GMAT课程,可加微信:jmtccca

BCE 选项,fossils 作主语,谓语动词应该用复数。


D 选项,正确。

951
Recently physicians have determined that stomach ulcers are not caused by stress, alcohol, or
rich foods, but a bacterium that dwells in the mucous lining of the stomach.

A.not caused by stress, alcohol, or rich foods, but

B.not caused by stress, alcohol, or rich foods, but are by

C.caused not by stress, alcohol, or rich foods, but by

D.caused not by stress, alcohol, and rich foods, but

E.caused not by stress, alcohol, and rich foods, but are by

解析:not…but…平行
C 选项,正确,not by…but by…

952
The eyes of the elephant seal adapt to darkness more quickly than any other animal vet tested,
thus allowing it to hunt efficiently under the gloomy conditions at its feeding depth of
between 300 and 700 meters.

A.The eyes of the elephant seal adapt to darkness more quickly than any other animal yet
tested, thus allowing it

B.The eyes of the elephant seal adapt to darkness more quickly than does any other animal
yet tested, allowing them

C.The eyes of the elephant seal adapt to darkness more quickly than do those of any other
animal yet tested, allowing it

D.Because they adapt to darkness more quickly than any other animal yet tested, the eyes of
the elephant seat allow it

E.Because the eyes of the elephant seal adapt to darkness more quickly than do those of any
other animal yet tested, it allows them

解析:A 选项:比较对象应该是“象海豹的眼睛适应黑暗的情况”和“其它动物的
眼睛适应黑暗的情况”,因此,than 后应改为:those of any other animal yet tested
do(those 表示 eyes,do 表示情况,即,adapt)。
B 选项:比较对象错误同 A;them 的指代对象只能是复数名词,但是逻辑上 allow

免费加入GMAT备考群、机经群,可加微信:jmtccca
免费领取GMAT资料、了解GMAT课程,可加微信:jmtccca

的宾语应该是 the elephant seal,是单数名词。


C 选项:正确。
D 选项:比较对象错误同 A。
E 选项:代词 it 和 them 的指代对象均有错误,错同 B。

953
A mutual fund having billions of dollars in assets will typically invest that money in hundreds
of companies, rarely holding more than one percent of the shares of any particular
corporation.

A.companies, rarely holding more than one percent

B.companies, and it is rare to hold at least one percent or more

C.companies and rarely do they hold more than one percent

D.companies, so that they rarely hold more than one percent

E.companies; rarely do they hold one percent or more

解析:holding more than one percent 是现在分词短语,做状语,表结果,其主语


和其所修饰句子的主语相同,即,a mutual fund;B 中的 it 为形式主语,正常语
序为 to hold at least one percent or more of the shares of any particular corporation is
rare,此时 to hold 部分为不定式作主语;CDE 中 hold 的主语均为 they(指代对
象为 assets)。
A 选项:正确。
B 选项:并列结构错误,逻辑上,“共同基金投钱进入数以百计的公司”这个事件
的结果是“共同基金在公司中占据股份”,因此需要分词做状语表结果。另外,at
least 和 or more 中有一个是赘余的,都表示了最低是 1%,需要删掉一个。
C 选项:代词 they 的指代对象只能为 assets 或者 companies。但在逻辑上,应该
是基金持有百分之一的股份而不是资产或公司们持有百分之一的股份;另外,
hold 句应做 invest 句的伴随状语,该错误同(B)。
D 选项:由 they 产生的逻辑主语错误同(C);hold 错误同(B)。
E 选项:由 they 产生的逻辑主语错误同(C);hold 错误同(B)。

954
Positing an enormous volcanic explosion at the end of the Permian period would explain the
presence of a buried crater, account for the presence of the element iridium (originating deep
within the earth), and the presence of quartz having been shattered by high impact shock
waves.

免费加入GMAT备考群、机经群,可加微信:jmtccca
免费领取GMAT资料、了解GMAT课程,可加微信:jmtccca

A.account for the presence of the element iridium (originating deep within the earth),
and the presence of quartz having been

B.of the element iridium (originating deep within the earth), and of quartz

C.the element iridium (originating deep within the earth), and explain the presence
of quartz having been

D.the presence of the element iridium (originating deep within the earth), and
explain the presence of quartz

E.explain the element iridium (originating deep within the earth), and the presence of
quartz

解析:考察平行,语义:定位一个爆炸可以解释一些现象的出现,这些现象包括
“埋在地下的陨石坑”,“铱元素(the element iridium)”,和“石英(quartz)”。三
者需要在语法上保持形式平行,B 选项正确。

955
The 19-year-old pianist and composer performed his most recent work all over Europe, Asia,
and North America last year, winning prestigious awards in both London as well as Tokyo for
his achievement at so young an age, and he is hoping to continue composing now that he has
returned to Chicago.

A.winning prestigious awards in both London as well as Tokyo for his achievement
at so young an age, and he is hoping

B.winning prestigious awards both in London and Tokyo for his achievement at such
a young age, and hoping

C.having won prestigious awards both in London and Tokyo for his achievement at
so young an age, hoping

D.winning prestigious awards in both London and Tokyo for his achievement at such
a young age, and he hopes

E.having won prestigious awards both in London as well as Tokyo for his
achievement at so young an age, and he hopes

解析:A B 选项:both…and…为习惯表达,both … as well as 搭配错误。


C 选项:both…and…不平行,另外,having won 应改为 winning。having done/having
been done 表示动作先发生,在逻辑上,不可能 win 这个动作先发生于 perform。

免费加入GMAT备考群、机经群,可加微信:jmtccca
免费领取GMAT资料、了解GMAT课程,可加微信:jmtccca

D 选项:正确。
E 选项:both 搭配错同 A;having won 错同 C。

956
Starfish, with anywhere from five to eight arms, have a strong regenerative ability, and if one
arm is lost it quickly replaces it, sometimes by the animal overcompensating and growing an
extra one or two.

A.one arm is lost it quickly replaces it, sometimes by the animal overcompensating and

B.one arm is lost it is quickly replaced, with the animal sometimes overcompensating and

C.they lose one arm they quickly replace it, sometimes by the animal overcompensating,

D.they lose one arm they are quickly replaced, with the animal sometimes
overcompensating,

E.they lose one arm it is quickly replaced, sometimes with the animal overcompensating,

解析:A 选项,it 指代单复数不一致,根据语义指代前面 starfish, fish 单复数同


行,根据谓语动词 have,starfish 是复数。
B 选项,正确。
C DE 选项,逻辑不合理,不可能是一群鱼丢一个 arm,应该是 they lose their arms。

957
In 2000, a mere two dozen products accounted for half the increase in spending on
prescription drugs, a phenomenon that is explained not just because of more expensive drugs
but by the fact that doctors are writing many more prescriptions for higher-cost drugs.

A.a phenomenon that is explained not just because of more expensive drugs but by the fact
that doctors are writing

B.a phenomenon that is explained not just by the fact that drugs are becoming more
expensive but also by the fact that doctors are writing

C.a phenomenon occurring not just because of drugs that are becoming more expensive but
because of doctors having also written

D.which occurred not just because drugs are becoming more expensive but doctors are also
writing

免费加入GMAT备考群、机经群,可加微信:jmtccca
免费领取GMAT资料、了解GMAT课程,可加微信:jmtccca

E.which occurred not just because of more expensive drugs but because doctors have also
written

解析:A 选项:not just…but(also)…不仅…而且…前后不平行,它们连接的两


者需要在语法结构上平行,即,not just by…but(also)by…或 not just because
of…but (also)because of…
B 选项:正确。
C 选项:核心语义错误,because of 是介词,其身后的 drugs that are becoming more
expensive 和 doctors having also written many more prescriptions for higher-cost
drugs 均为名词短语,这两个名词短语的核心词分别是 drugs 和 doctors。在逻辑
上,原因应该是“药物变贵”和“医生开处方”这两件事,而不是药和医生这两个名
词。
D 选项:which occurred 是非限定性定语从句,就近修饰先行词 prescription drugs
不合理,which 不具有修饰一句话的功能。
E 选项:which occurred 错同 D。

958
Similar to other early Mississippi Delta blues singers, the music of Robert Johnson arose from
an oral tradition beginning with a mixture of chants, fiddle tunes, and religious music and
only gradually evolved into the blues.

A.Similar to other early Mississippi Delta blues singers, the music of Robert Johnson arose
from an oral tradition beginning with

B.Similar to that of other early Mississippi Delta blues singers, Robert Johnson made music
that arose from an oral tradition that began with

C.As with other early Mississippi Delta blues singers, Robert Johnson made music that arose
from an oral tradition beginning as

D.Like other early Mississippi Delta blues singers, Robert Johnson’s music arose from an
oral tradition beginning with

E.Like the music of other early Mississippi Delta blues singers, the music of Robert Johnson
arose from an oral tradition that began as

解析:考察比较对象的可比性,比较对象要么都是人,要么都是音乐,不能用人
和音乐比较,排除(A)(B)(C)(D),答案为(E)。

959
Thelonious Monk, who was a jazz pianist and composer, produced a body of work both
rooted in the stride-piano tradition of Willie (The Lion) Smith and Duke Ellington, yet in
many ways he stood apart from the mainstream jazz repertory.

免费加入GMAT备考群、机经群,可加微信:jmtccca
免费领取GMAT资料、了解GMAT课程,可加微信:jmtccca

A.Thelonious Monk, who was a jazz pianist and composer, produced a body of work
both rooted

B.Thelonious Monk, the jazz pianist and composer, produced a body of work that
was rooted both

C.Jazz pianist and composer Thelonious Monk, who produced a body of work rooted

D.Jazz pianist and composer Thelonious Monk produced a body of work that was
rooted

E.Jazz pianist and composer Thelonious Monk produced a body of work rooted both

解析:A B E 选项,both…and…不平行,C 选项,句子无谓语。

960
Nobody knows exactly how many languages there are in the world, partly because of the
difficulty of distinguishing between a language and the sublanguages or dialects within it, but
those who have tried to count typically have found about five thousand.

A.and the sublanguages or dialects within it, but those who have tried to count typically have
found

B.and the sublanguages or dialects within them, with those who have tried counting typically
finding

C.and the sublanguages or dialects within it, but those who have tried counting it typically
find

D.or the sublanguages or dialects within them, but those who tried to count them typically
found

E.or the sublanguages or dialects within them, with those who have tried to count typically
finding

解析:A 选项,正确。根据语义,在一门语言下的子语言和方言,it 指代 a language


单复数不一致。排除 B D E 选项。C 选项,try doing 错误,try to do 表示某人尝
试去做某事(总体感觉很吃力,并且通常要有结果),而 try doing 表示试着做
(总体感觉很轻松,不需要有结果),根据语义这里要用 try to do。

961
Although a number of excellent studies narrate the development of domestic technology and
its impact on housewifery, these works do not discuss the contributions of the women

免费加入GMAT备考群、机经群,可加微信:jmtccca
免费领取GMAT资料、了解GMAT课程,可加微信:jmtccca

employed by manufacturers and utility companies as product demonstrators and


publicists, who initially promoted new and unfamiliar technology to female consumers.

A.by manufacturers and utility companies as product demonstrators and publicists,

B.to be product demonstrators and publicists by manufacturers and utility companies,

C.to demonstrate and publicize their products by manufacturers and utility companies

D.by manufacturers and utility companies to be demonstrators and publicists of their


products

E.by manufacturers and utility companies to demonstrate and publicize their products

解析:考察就近修饰,demonstrators and publicists 去 promote,who 定从就近修


饰合理,A 选项,正确。

962
The absence from business and financial records of the nineteenth century of statistics about
women leave us with no record of the jobs that were performed by women and how they
survived economically.

A.from business and financial records of the nineteenth century of statistics about women
leave us with no record of the jobs that were performed by women and

B.from business and financial records of statistics about women from the nineteenth century
leave us with no record of what jobs women performed or

C.of statistics for women from business and financial records in the nineteenth century
leaves us with no record of either the jobs that women were performing and of

D.of statistics on women from business and financial records in the nineteenth century leave
us with no record of the jobs that women performed or of

E.of statistics about women from business and financial records of the nineteenth century
leaves us with no record of either what jobs women performed or

解析:ABD 选项:主语是 absence,谓语动词也应该用单数,即,leaves。


C 选项:either…or…固定搭配,either…and…错误。
E 选项:正确。

963

免费加入GMAT备考群、机经群,可加微信:jmtccca
免费领取GMAT资料、了解GMAT课程,可加微信:jmtccca

Heating-oil prices are expected to be higher this year than last because refiners are paying
about $5 a barrel more for crude oil than they were last year.

A.Heating-oil prices are expected to be higher this year than last because refiners are paying
about $5 a barrel more for crude oil than they were

B.Heating-oil prices are expected to rise higher this year over last because refiners pay about
$5 a barrel for crude oil more than they did

C.Expectations are for heating-oil prices to be higher this year than last year’s because
refiners are paying about $5 a barrel for crude oil more than they did

D.It is the expectation that heating-oil prices will be higher for this year over last because
refiners are paying about $5 a barrel more for crude oil now than what they were

E.It is expected that heating-oil prices will rise higher this year than last year’s because
refiners pay about $5 a barrel for crude oil more than they did

解析:A 选项:正确。比较结构主干相同,留下不同的时间状语 this year 和 last year。


B 选项:rise 表达了“上升”的意思,而 higher 表示更高。在逻辑上,rise 的结果
一定是“使得某物 higher”,因此,rise 或 higher 只要存在一个; 另外, higher…than…
固定搭配,higher…over…错误。
C 选项:比较对象不对等。比较两个年份的价格在本题中也是可以的,但是,last
year's 在 than 的身前无法找到平行对象,可以改为:this year's heating-oil prices are
higher than last year's prices.
D 选项:higher…over…错同 B。
E 选项:错同 B。

964
Even though Clovis points, spear points with longitudinal grooves chipped onto their faces,
have been found all over North America, they are named for the New Mexico site where they
were first discovered in 1932.

A.Even though Clovis points, spear points with longitudinal grooves chipped onto their faces,
have been found all over North America, they are named for the New Mexico site where they
were first discovered in 1932.

B.Although named for the New Mexico site where first discovered in 1932, Clovis points
are spear points of longitudinal grooves chipped onto their faces and have been found all over
North America.

免费加入GMAT备考群、机经群,可加微信:jmtccca
免费领取GMAT资料、了解GMAT课程,可加微信:jmtccca

C.Named for the New Mexico site where they have been first discovered in 1932, Clovis
points, spear points of longitudinal grooves chipped onto the faces, have been found all over
North America.

D.Spear points with longitudinal grooves that are chipped onto the faces, Clovis points, even
though named for the New Mexico site where first discovered in 1932, but were found all
over North America.

E.While Clovis points are spear points whose faces have longitudinal grooves chipped into
them, they have been found all over North America, and named for the New Mexico site
where they have been first discovered in 1932.

解析:A 选项: 正确。


BC 选项:未遵从原句合理的逻辑语义,逻辑关系倒置。
D 选项:没有谓语动词。
E 选项:转折关系不合理。

965
Heavy commitment by an executive to a course of action, especially if it has worked well in
the past, makes it likely to miss signs of incipient trouble or misinterpret them when they do
appear.

A.Heavy commitment by an executive to a course of action, especially if it has worked well


in the past, makes it likely to miss signs of incipient trouble or misinterpret them when they
do appear.

B.An executive who is heavily committed to a course of action, especially one that worked
well in the past, makes missing signs of incipient trouble or misinterpreting ones likely when
they do appear.

C.An executive who is heavily committed to a course of action is likely to miss or


misinterpret signs of incipient trouble when they do appear, especially if it has worked well in
the past.

D.Executives’ being heavily committed to a course of action, especially if it has worked well
in the past, makes them likely to miss signs of incipient trouble or misinterpreting them when
they do appear.

E.Being heavily committed to a course of action, especially one that has worked well in the
past, is likely to make an executive miss signs of incipient trouble or misinterpret them when
they do appear.

免费加入GMAT备考群、机经群,可加微信:jmtccca
免费领取GMAT资料、了解GMAT课程,可加微信:jmtccca

解析:语义:对一项行动方针,尤其是过去行之有效的行动方针,投入过多,很
可能会让一位高管错过一些初期问题的迹象,或者在它们确实出现时,对它们产
生误解。
A 选项,it 特指,a course of action,泛指应使用 one。
BC 选项,语义重心错误,原句是说的过度投入会使错失 signs 变得可能,而 BC
是说 An executive 会使错失 signs 变得可能。
D 选项,make them likely 句型错误,形式宾语要用 it。
E 选项,正确。

966
According to recent studies comparing the nutritional value of meat from wild animals and
meat from domesticated animals, wild animals have less total fat than do livestock fed on
grain and more of a kind of fat they think is good for cardiac health.

A.wild animals have less total fat than do livestock fed on grain and more of a kind of fat
they think is

B.wild animals have less total fat than livestock fed on grain and more of a kind of fat
thought to be

C.wild animals have less total fat than that of livestock fed on grain and have more fat of a
kind thought to be

D.total fat of wild animals is less than livestock fed on grain and they have more fat of a
kind thought to be

E.total fat is less in wild animals than that of livestock fed on grain and more of their fat is of
a kind they think is

解析:此题两个比较对象,野生动物和家畜,比动词,动词一般不可省。原文逻
辑没问题,但原文 they 无指代,所以保留原文逻辑,改语法错误即可。
A.错。原文语义没问题,than do 表达正确,但 They 无指代。
B.正确。虽然 than 后面省略了助动词,但 B 还是正确,为什么呢,因为除了脂
肪财富这俩属性的助动词可以省之外,其他时候比动词,动词一般不可省。此题
出题人认为不可能理解成野生动物有家畜这层歧义,只可能理解成野生动物和家
畜分别拥有脂肪,所以 B 即便是省了助动词,大结构大语义没问题且综合其他
选项的明显错误,仍然可选。
C.错。that of 指脂肪,导致 wild animals 和 that of 不可比,后面措辞 fat of a kind
不符合表达习惯,应该是 kind of fat.
D.错。total fat of 和 livestock 不可比,fat of a kind 不符合表达习惯。
E.错。more of their fat is of a kind they think is 不简洁。

免费加入GMAT备考群、机经群,可加微信:jmtccca
免费领取GMAT资料、了解GMAT课程,可加微信:jmtccca

967
Yellow jackets number among the 900 or so species of the world’s social wasps, wasps living
in a highly cooperative and organized society where they consist almost entirely
of females—the queen and her sterile female workers.

A.wasps living in a highly cooperative and organized society where they consist almost
entirely of

B.wasps that live in a highly cooperative and organized society consisting almost entirely of

C.which means they live in a highly cooperative and organized society, almost all

D.which means that their society is highly cooperative, organized, and it is almost entirely

E.living in a society that is highly cooperative, organized, and it consists of almost all

解析:A 选项:复数代词 they 在语法上只能指代复数名词 wasps,但是在逻辑上,


由女王和工人组成的应该是某一个社会(society)而不是 wasps。
B 选项:正确。
CD 选项:which means 引导定从,定从不具有修饰一句话的功能。
E 选项:and it,it 多余 。改:in a society that is highly cooperative, organized, and
consists of almost all.

968
Before 1988, insurance companies in California were free to charge whatever rates the market
would bear, needing no approval from regulators before raising rates.

A.needing no approval from regulators before raising

B.and it needed no approval by regulators before raising

C.and needing no approval from regulators before they raised

D.with approval not needed by regulators before they raised

E.with no approval needed from regulators before the raising of

解析:语义:在 1988 之前,加利福尼亚的保险公司自由地去收取费用,不需要


任何的批准。
A 选项:正确,needing 的逻辑主语是 insurance companies。
B 选项:代词 it 的指代对象必须为单数名词,但在逻辑上,真正发出 need 这个
动作的应该是保险公司们(insurance companies),指代单复数不一致。
C 选项:and needing 不平行。

免费加入GMAT备考群、机经群,可加微信:jmtccca
免费领取GMAT资料、了解GMAT课程,可加微信:jmtccca

DE 选项:逻辑上是保险公司发出“需要”这一动作,即,保险公司不需要从规定
者的手里获取批准,这里并没有指明是谁发出 need 这一动作,表义不清。

969

解析:首先原文逻辑就是错的,根据语义这里的 a tool for private conversation 不


修饰电话,应该是 radio 的同位语,排除 ABE。
D 选项,which 定从修饰错误。
C 选项,that 从句跳跃修饰 radio。

970
Because there are provisions of the new maritime code that provide that even tiny islets can
be the basis for claims to the fisheries and oil fields of large sea areas, they have already
stimulated international disputes over uninhabited islands.

A.Because there are provisions of the new maritime code that provide that even tiny islets
can be the basis for claims to the fisheries and oil fields of large sea areas, they have already
stimulated

B.Because the new maritime code provides that even tiny islets can be the basis for claims to
the fisheries and oil fields of large sea areas, it has already stimulated

C.Even tiny islets can be the basis for claims to the fisheries and oil fields of large sea areas
under provisions of the new maritime code, already stimulating

D.Because even tiny islets can be the basis for claims to the fisheries and oil fields of large
sea areas under provisions of the new maritime code, this has already stimulated

E.Because even tiny islets can be the basis for claims to the fisheries and oil fields of large
sea areas under provisions of the new maritime code, which is already stimulating

解析: 语义:因为新的海上条约要求即使小岛屿也能当作渔业和油田的声索依
据,它已经激发了国际上对无人岛的争端。
A 选项,太啰嗦了 there are provisions 和 provide 重复了;硬伤:they 指代 code
单数错误。
B 选项,正确。

C 选项, 重心改变,修饰成分:“under provisions”本意上是主谓结构“Even tiny islets


can be the basis……”的状语,但由于其位于主谓结构的句尾且未与主谓结构之间
并为以逗号相隔,因此其有作定语修饰“sea areas”表达“在新海上条约要求下的海
域”的意思的可能,因此修饰关系并不明确。
D 选项, this 裸奔。
E 选项,只有从句没有主句。

免费加入GMAT备考群、机经群,可加微信:jmtccca
免费领取GMAT资料、了解GMAT课程,可加微信:jmtccca

971
Unlike the automobile company, whose research was based on crashes involving sport utility
vehicles, the research conducted by the insurance company took into account such factors as a
driver's age, sex, and previous driving record.

A.company, whose research was based on

B.company, which researched

C.company, in its research of

D.company's research, having been based on

E.company's research on

解析:ABC 选项:company 和 research 比较对象不可比。


D 选项:比较对象虽然是对等的,但是 having been based on crashes 在本选项中
更像是一个伴随状语,用来修饰整个题目的主句。这显然是不符合逻辑的,该部
分在逻辑上应修饰 company。
E 选项:正确。

972
Gusty westerly winds will continue to usher in a seasonably cool air mass into the region, as a
broad area of high pressure will build and bring fair and dry weather for several days.

A.to usher in a seasonably cool air mass into the region, as a broad area of high
pressure will build and

B.ushering in a seasonably cool air mass into the region and a broad area of high
pressure will build that

C.to usher in a seasonably cool air mass to the region, a broad area of high pressure
building, and

D.ushering a seasonably cool air mass in the region, with a broad area of high
pressure building and

E.to usher a seasonably cool air mass into the region while a broad area of high
pressure builds, which will

解析:主将从现是指在以 if(如果),when,as,while 等引导的时间状语从句和条


件状语从句中,如果主句是一般将来时,从句用一般现在时替代一般将来时
Usher...而不是 usher into/in....ABC 冗余。

免费加入GMAT备考群、机经群,可加微信:jmtccca
免费领取GMAT资料、了解GMAT课程,可加微信:jmtccca

A 选项,AS 做状语,当...时,要用主将从现。
B 选项,svo and svo 错误,句子结构不合理。
C 选项,高压变成了插入语,逻辑不合理 主干为带来冷空气和干燥?
D 选项,and 前后不平行。
E 选项,正确,which 指代 a broad area of high pressure,依然指代名词,只不过
跳跃了动词 builds,符合主将从现。

973
With the patience of its customers and with its network strained to the breaking point, the
on-line service company announced a series of new initiatives trying to relieve the congestion
that has led to at least four class-action lawsuits and thousands of complaints from frustrated
customers.

A.the patience of its customers and with its network strained to the breaking point, the
on-line service company announced a series of new initiatives trying to relieve

B.the patience of its customers and its network strained to the breaking point,the on-line
service company announced a series of new initiatives that try to relieve

C.its network and the patience of its customers strained to the breaking point, the on-line
service company announced a series of new initiatives to try to relieve

D.its network and with the patience of its customers strained to the breaking point, the
on-line service company announced a series of initiatives to try relieving

E.its network and its customers' patience strained to the breaking point, the on-line service
company announced a series of new initiatives to try relieving

解析:AD 选项,With the patience of its customers and with its network strained to the
breaking point,and 前面的 with 结构语义不完整,原句要表达的意思应该是,因
为客户的耐心和网络都已经到了崩溃的边缘,网络公司宣布了一系列新政策去缓
解...,不需要中间再来个 with 把语义隔开。
B 选项是前面平行的语义问题 the patience (of its customers and its network 修饰
部分)network 是不会有耐心的。
C 选项,正确,独立主格 With (its network and the patience of its customers 逻辑
主语) strained to the breaking point 逻辑谓语。
try to do 的意思是努力去做某事,侧重于为了达到目的而努力。
try doing 只是一种新的尝试,不带目的性,成不成功无所谓。
根据语义这里需要的是 try to do 的语义。

974
November is traditionally the strongest month for sales of light trucks, but sales this past
November, even when compared with sales in previous Novembers, accounted for a
remarkably large share of total vehicle sales.

免费加入GMAT备考群、机经群,可加微信:jmtccca
免费领取GMAT资料、了解GMAT课程,可加微信:jmtccca

A.but sales this past November, even when compared with sales in previous
Novembers,

B.but even when it is compared with previous Novembers, this past November’s
sales

C.but even when they are compared with previous Novembers, sales of light trucks
this past November

D.so that compared with previous Novembers, sales of light trucks this past
November

E.so that this past November’s sales, even compared with previous Novembers’
sales,

解析:这道题目首先要把句子看懂,这句子的意思是:11 月传统来讲是卖的最
好的月份,但是今年的 11 月份,卖的传统的还要好很多。
A 选项,正确,可比。
B 选项,when it 中的 it 是谁?主句的主语是 sales,sales 和 precious November
比?It 本意是想指代 sales,但是 sales 是复数,也不行。past November`s sales 中
的刚过去的十一月是定语!不能被指带,主句的主语是 sales。以前十一月份的
销量 和 这个十一月的销量 才可比,不能是以前的十一月和这个十一月的销量
比 不可比。
C 选项,it 换成了 they,sales 和之前的月份 不可比,如果要指代 this past
November,应该用 it。
D 选项,本意要强调的是特殊性,不应该用 so that,没有这种关系,不可比。
E 选项,可比,纠结的话应该是 AE 之间选,注意作者的 intended meaning 是强
调,而不是 so that,还要注意更优选不用所有格,人或者动物用所有格较好!比
如 the month of festival 而不是 festival’s month。

975
Most of the county’s biggest daily newspapers had lower circulation in the six months from
October 1995 through March 1996 than a similar period a year earlier.

A.a similar period

B.a similar period's

C.in a similar period

D.that in a similar period

E.that of a similar period

免费加入GMAT备考群、机经群,可加微信:jmtccca
免费领取GMAT资料、了解GMAT课程,可加微信:jmtccca

解析:主干相同,留下不同的时间状语比较,in the six months 和 in a similar period...


比较哦,不同时期下一个销量的对比。

976
Mauritius was a British colony for almost 200 years, excepting for the domains of
administration and teaching, the English language was never really spoken on the island.

A.excepting for

B.except in

C.but except in

D.but excepting for

E.with the exception of

解析:前后两个句子,中间需要连接词,且前后两个句子是转折关系,排除 ABE
选项;except(English language)in the domains...,排除 D 选项。

977
Although appearing less appetizing than most of their round and red supermarket cousins,
heirloom tomatoes, grown from seeds saved during the previous year--they are often green
and striped, or have plenty of bumps and bruises—heirlooms are more flavorful and thus in
increasing demand.

A.Although appearing less appetizing than most of their round and red supermarket cousins,
heirloom tomatoes, grown from seeds saved during the previous year

B.Although heirloom tomatoes, grown from seeds saved during the previous year, appear
less appetizing than most of their round and red supermarket cousins

C.Although they appear less appetizing than most of their round and red supermarket
cousins, heirloom tomatoes, grown from seeds saved during the previous year

D.Grown from seeds saved during the previous year, heirloom tomatoes appear less
appetizing than most of their round and red supermarket cousins

E.Heirloom tomatoes, grown from seeds saved during the previous year, although they
appear less appetizing than most of their round and red supermarket cousins

免费加入GMAT备考群、机经群,可加微信:jmtccca
免费领取GMAT资料、了解GMAT课程,可加微信:jmtccca

解析:ACE 选项: S,SVO 结构错误。


B 选项:正确。
D 选项:没有让步关系,没有 although 这个连词,直接导致 run on。
句子结构:双破折号属于插入语,添加之后对其他部分的句子结构不造成任何影
响。因此将这一部分删去后考察句子结构,发现“heirloom tomatoes appear less
appetizing……”与“heirlooms are more flavorful……”两个主谓结构之间没有任何
分句连接词,因此这两个结构的连接方式有问题。

978
The World Wildlife Fund has declared that global warming, a phenomenon most scientists
agree to be caused by human beings in burning fossil fuels, will create havoc among
migratory birds by altering the environment in ways harmful to their habitats.
A.a phenomenon most scientists agree to be caused by human beings in burning fossil fuels,

B.a phenomenon most scientists agree that is caused by fossil fuels burned by human beings,

C.a phenomenon that most scientists agree is caused by human beings’ burning of fossil
fuels,

D.which most scientists agree on as a phenomenon caused by human beings who burn fossil
fuels,

E.which most scientists agree to be a phenomenon caused by fossil fuels burned by human
beings,

解析:A 选项,agree 不及物 V, a phenomenon most scientists agree(a phenomenon)


to be caused by human beings in burning fossil fuels,把定从还原,先行词做 agree
的宾语,需要 on 或 about。
B 选项,a phenomenon( most scientists agree ) (that is caused by fossil fuels burned
by human beings),这个选项就是,科学家同意这个被...导致的现象,语义不合
理 ,且 that 跳跃修饰不优选。
C 选项,a phenomenon that most scientists agree(that a phenomenon ) is caused by
human beings’ burning of fossil fuels,that 定从修饰现象,先行词在定从中做定语
从句中宾语从句的主语,这里是省略 that 的宾从。语义:一个现象,什么现象呢,
定从修饰,就是由人类燃烧...导致的一个现象。
DE 选项,which 不修饰一句话。

979
The largest of all the planets, not only is Jupiter three times so massive as Saturn, the next
larger planet, but also possesses four of the largest satellites, or moons, in our solar system.

A.not only is Jupiter three times so massive as Saturn, the next larger

免费加入GMAT备考群、机经群,可加微信:jmtccca
免费领取GMAT资料、了解GMAT课程,可加微信:jmtccca

B.not only is Jupiter three times as massive as Saturn, the next largest

C.Jupiter, not only three times as massive as Saturn, the next largest

D.Jupiter not only is three times as massive as Saturn, the next largest

E.Jupiter is not only three times so massive as Saturn, the next larger

解析:not only but also 应该是要平行的,but also 后面未划线,接的是 possess


谓语。AB 选项,后面接的是 Jupiter is,主谓结构的倒装,不平行。
C 选项,Jupiter 作为主语,没有谓语???而且还不平行。
D 选项正确,平行。
E 选项,不平行。
且 as as 还是 so as,如果是否定句,这里用 so as 是可以的,so as 只能用于否定
句,是 as as 的否定形式;the next largest 还是 the next larger 应该用最高级,因
为即便是第二大 也是和剩下的所有去比较 larger 的话,和谁比 larger?第一大,
第二大,第三大。

980
While many of the dinosaur fossils found recently in northeast China seem to provide
evidence of the kinship between dinosaurs and birds, the wealth of enigmatic fossils seem
more likely at this stage that they will inflame debates over the origin of birds rather
than settle them.

A.seem more likely at this stage that they will inflame debates over the origin of
birds rather than

B.seem more likely that it will inflame debates over the origin of birds at this stage
than

C.seems more likely to inflame debates on the origin of birds at this stage rather than

D.seems more likely at this stage to inflame debates over the origin of birds than to

E.seems more likely that it will inflame debates on the origin of birds at this stage
than to

解析: 考察平行,主谓一致。主句主语核心词是 wealth,谓语动词应为 seems,


排除 AB;C 选项,more...rather than 用法错误;D 选项,正确;E 选项,that it will
inflame 和 to settle 不平行。

981

免费加入GMAT备考群、机经群,可加微信:jmtccca
免费领取GMAT资料、了解GMAT课程,可加微信:jmtccca

Found only in the Western Hemisphere and surviving through extremes of climate,
hummingbirds’ range extends from Alaska to Tierra del Fuego, from sealevel rain forests to
the edges of Andean snowfields and ice fields at altitudes of 15,000 feet.

A.Found only in the Western Hemisphere and surviving through extremes of climate,
hummingbirds’ range extends

B.Found only in the Western Hemisphere, hummingbirds survive through extremes of


climate, their range extending

C.Hummingbirds, found only in the Western Hemisphere and surviving through extremes of
climate, with their range extending

D.Hummingbirds, found only in the Western Hemisphere and surviving through extremes of
climate, their range extends

E.Hummingbirds are found only in the Western Hemisphere, survive through extremes of
climate, and their range extends

解析:前置状语 found...and surviving...,逻辑主语是 range,鸟类是可以被发现和


生存的,但是 ranges 是不能和 surviving 搭配的,所以 A 选项错误;B 选项,正
确;C 选项,缺主语;D 选项有两个主语;E 选项有两个谓语,但是缺连词连接,
逗号不能连接两个句子,不能连接两个动词。

982
She was less successful after she had emigrated to New York compared to her native Germany,
photographer Lotte Jacobi nevertheless earned a small group of discerning admirers, and her
photographs were eventually exhibited in prestigious galleries across the United States.

A.She was less successful after she had emigrated to New York compared to

B.Being less successful after she had emigrated to New York as compared to

C.Less successful after she emigrated to New York than she had been in

D.Although she was less successful after emigrating to New York when compared to

E.She had been less successful after emigrating to New York than in

解析:ABD 选项,less...(as/when)compared to,搭配错误。


E 选项,逗号不能连接两个句子。

983

免费加入GMAT备考群、机经群,可加微信:jmtccca
免费领取GMAT资料、了解GMAT课程,可加微信:jmtccca

Scientists have recently found evidence that black holes—regions of space in which matter is
so concentrated and the pull of gravity so powerful that nothing, not even light, can emerge
from them—probably exist at the core of nearly all galaxies and the mass of each black hole
is proportional to its host galaxy.

A.exist at the core of nearly all galaxies and the mass of each black hole is proportional to

B.exist at the core of nearly all galaxies and that the mass of each black hole is proportional
to that of

C.exist at the core of nearly all galaxies, and that the mass of each black hole is proportional
to

D.exists at the core of nearly all galaxies, and that the mass of each black hole is
proportional to that of

E.exists at the core of nearly all galaxies and the mass of each black hole is proportional to
that of
解析:that 同位语从句的主语是 black holes,DE 选项,主谓单复数不一致;
AC 选项,比较对象不可比;B 选项,正确。

984
The use of lie detectors is based on the assumption that lying produces emotional reactions in
an individual that, in turn, create unconscious physiological responses.

A.that, in turn, create unconscious physiological responses

B.that creates unconscious physiological responses in turn

C.creating, in turn, unconscious physiological responses

D.to create, in turn, physiological responses that are unconscious

E.who creates unconscious physiological responses in turn

解析:A 选项,正确。BE 选项就近修饰 individual,语义不合理,这里联系前后


语义,是 emotional reactions in turn create physical reactions,情绪反应进一步产生
下意识的生理反应,比如撒谎紧张导致脸红结巴;且 B 选项 in turn 位置错误,
根据语义是修饰 create 的,进而产生…;CD 选项,creating/to create 修饰歧义,
是修饰 individual 还是 reactions,而 that 定从里面是 create 复数,那么可以确定
是修饰 reactions,不会有歧义。

免费加入GMAT备考群、机经群,可加微信:jmtccca
免费领取GMAT资料、了解GMAT课程,可加微信:jmtccca

985
Australian embryologists have found evidence that suggests that the elephant is descended
from an aquatic animal, and its trunk originally evolving as a kind of snorkel.
A.that suggests that the elephant is descended from an aquatic animal, and its trunk
originally evolving

B.that has suggested the elephant descended from an aquatic animal, its trunk originally
evolving

C.suggesting that the elephant had descended from an aquatic animal with its trunk
originally evolved

D.to suggest that the elephant had descended from an aquatic animal and its trunk originally
evolved

E.to suggest that the elephant is descended from an aquatic animal and that its trunk
originally evolved

解析:E 选项,正确。elephant's trunk 它的鼻子最初是作为一种浮潜器进化的,


后面第一个从句中讲的是客观事实,用一般现在时表示,第二个从句讲的过去事
实,用一般过去时,排除 BCD 选项,A 选项,and 不平行。

986
Most efforts to combat such mosquito-borne diseases like malaria and dengue have focused
either on the vaccination of humans or on exterminating mosquitoes with pesticides.

A.like malaria and dengue have focused either on the vaccination of humans or on
exterminating

B.like malaria and dengue have focused either on vaccinating of humans or on the
extermination of

C.as malaria and dengue have focused on either vaccinating humans or on exterminating

D.as malaria and dengue have focused on either vaccinating of humans or on extermination
of

E.as malaria and dengue have focused on either vaccinating humans or exterminating

解析:AB 选项:such…like…搭配错误。
CD 选项:either…or…不平行。
E 选项:正确。

免费加入GMAT备考群、机经群,可加微信:jmtccca
免费领取GMAT资料、了解GMAT课程,可加微信:jmtccca

987
Among the Tsonga, a Bantu-speaking group of tribes in southeastern Africa, dance teams
represent their own chief at the court of each other,providing entertainment in return for food,
drink, and lodging.

A.the court of each other, providing entertainment in return for

B.the court of another and provide entertainment in return for

C.the court of the other, so as to provide entertainment as a return on

D.each other's court,entertainmentbeing provided in return for

E.another's court and provide entertainment as a return on

解析:语义:一群在非洲东南部讲班图语的特松加部落中,舞蹈队代表自己的首
领在另一个部落的宫廷中表演,以换取食物、饮料和住宿。
解析:这里注意 a group of,一群这样的部落,所以可以直接排除 each other、the
other,这种数量背景是两者间的选项,即 ACD 选项。
E 选项,as a return on 用词搭配错误。
其次:在两个选项都没有其它语法错误,语义表达合理一致的情况下,of 所有
格>N’s,所以用词搭配没有积累到话,也可以通过优选排除 E 选项。

988
Almost like clones in their similarity to one another, the cheetah species’ homogeneity makes
them especially vulnerable to disease.

A.the cheetah species’ homogeneity makes them especially vulnerable to disease

B.the cheetah species is especially vulnerable to disease because of its homogeneity

C.the homogeneity of the cheetah species makes it especially vulnerable to disease

D.homogeneity makes members of the cheetah species especially vulnerable to disease

E.members of the cheetah species are especially vulnerable to disease because of their
homogeneity

解析:ACD 选项:和克隆生物比的肯定是猎豹,而不能是基因同质性这个概念。
B 选项:its homogeneity 单数逻辑错误,两者才能相同比较。
E 选项:正确。

免费加入GMAT备考群、机经群,可加微信:jmtccca
免费领取GMAT资料、了解GMAT课程,可加微信:jmtccca

989
As sources of electrical power, windmills now account for only about 2,500 megawatts
nationwide, but production is almost expected to double by the end of the year, which would
provide enough electricity for 1.3 million households.

A.almost expected to double by the end of the year, which would provide

B.almost expected that it will double by the end of the year, thus providing

C.expected that it will almost double by the end of the year to provide

D.expected almost to double by the end of the year and thus to provide

E.expected almost to double by the end of the year, which would thus be providing

解析:AB 选项 almost 副词的位置错误导致逻辑错误,almost expected 表达几乎


被预测,应该是“几乎要翻倍”。
C 选项,除了表达啰嗦之外,to provide 表达目的语义相比于 D 也不佳,to provide
并不是 double 的目的,而应该是结果。
D 选项,正确。
E 选项,which 修饰有歧义,而且时态用 would be doing 错误,应该用 will do。

990
While most of the earliest known ball courts in Mesoamerica date to 900-400 B.
C., waterlogged latex balls found at El Manati and representations of ballplayers painted on
ceramics found at San Lorenzo attest to the fact that the Mesoamerican ballgame was well
established by the mid-thirteenth century B.C.

A.waterlogged latex balls found at EI Manati and representations of ballplayers painted on


ceramics found at San Lorenzo attest

B.waterlogged latex balls found at EI Manati and the painting of representations of


ballplayers on ceramics found at San Lorenzo attests

C.waterlogged latex balls found at EI Manati and ceramics painted with representations of
ballplayers found at San Lorenzo attests

D.the finding of waterlogged latex balls at EI Manati and the painting of representations of
ballplayers on ceramics found at San Lorenzo attests

E.the finding of waterlogged latex balls at EI Manati and of representations of ballplayers


painted on ceramics at San Lorenzo attest

免费加入GMAT备考群、机经群,可加微信:jmtccca
免费领取GMAT资料、了解GMAT课程,可加微信:jmtccca

解析:语义:虽然大多数中美洲的最早的球法院可以追溯到公元前 900 - 400 年,


(在 EM 被发现的)wlball 和 representations(在 C 陶瓷{在 SL 被发现的}上画的
球员的)描绘证明了美洲球赛在 13 世纪中叶被建立。
A 选项,正确,语义合理,球和描绘的内容两个证据证明了美洲球赛建立的时间。
B 选项,the painting of representations of ballplayers 语义错误,不是 painting 本身
证实...是画中具体对 ballplayers 描绘的内容证实...
C 选项,and ceramics 语义错同 B,且 found 修饰错误,不是 ballplayer 被发现
且 BC 选项,主谓单复数不一致。
DE 选项,the finding...,核心语义错误。

991
As criminal activity on the Internet becomes more and more sophisticated, not only are
thieves able to divert cash from company bank accounts, they can also pilfer valuable
information such as business development strategies, ne w product specifications, and
contract bidding plans, and sell the data to competitors.

A.they can also pilfer valuable information such as business development strategies, new
product specifications, and contract bidding plans, and sell

B.they can also pilfer valuable information that includes business development strategies, new
product specifications, and contract bidding plans, and selling

C.also pilfering valuable information including business development strategies, new product
specifications, and contract bidding plans, selling

D.but also pilfer valuable information such as business development strategies, new product
specifications, and contract bidding plans to sell

E.but also pilfering valuable information such as business development strategies, new
product specifications, and contract bidding plans and selling

解析:考察 not only...but also...平行,but also 二者可以省略其一


Not only 置于句首部分倒装,即 not only 后接的是一个句子,but also 接相同的结
构,省略了 but,后面 and sell 和前面 pilfer 平行。选择 A 选项。

992
Last week local shrimpers held a news conference to take some credit for the resurgence of
the rare Kemp’s ridley turtle, saying that their compliance with laws requiring that
turtle-excluder devices be on shrimp nets protect adult sea turtles.

A.requiring that turtle-excluder devices be on shrimp nets protect

B.requiring turtle-excluder devices on shrimp nets is protecting

免费加入GMAT备考群、机经群,可加微信:jmtccca
免费领取GMAT资料、了解GMAT课程,可加微信:jmtccca

C.that require turtle-excluder devices on shrimp nets protect

D.to require turtle-excluder devices on shrimp nets are protecting

E.to require turtle-excluder devices on shrimp nets is protecting

解析:saying that 宾从的主语是 compliance,ACD 选项,主谓单复数不一致。E


选项,to require 做后置定语修饰 laws 语义不合理,根据语义这个法律已经在实
施中了,所以不用 to do 不定式修饰。

993
A ruined structure found at Aqaba, Jordan, was probably a church, as indicated in its eastward
orientation and by its overall plan, as well as artifacts, such as glass oil-lamp fragments, found
at the site.

A.A ruined structure found at Aqaba, Jordan, was probably a church, as indicated in its
eastward orientation and by its overall plan, as well as

B.A ruined structure found at Aqaba, Jordan, once probably being a church, was indicated by
its eastward orientation, overall plan, and

C.Indicating that a ruined structure found at Aqaba, Jordan, was probably a church were its
eastward orientation and overall plan, but also the

D.A ruined structure found at Aqaba, Jordan, was probably a church, as indicates its
eastward orientation and overall plan, as well as the

E.That a ruined structure found at Aqaba, Jordan, was probably a church is indicated by its
eastward orientation and overall plan, as well as by the

解析:A 选项:in its eastward orientation 介词错误,被动应用 by 表示。


B 选项:was indicated 的主语 A ruined structure( found at Aqaba, Jordan, once
probably being a church 修饰),但在逻辑上,被计划和朝向表明的应该是“一个已
毁坏的结构可能是一座教堂”这个事件,而不是“一个已毁坏的结构本身”。
C 选项:主语是一个动名词短语,即,indicating that a ruined structure found at
Aqaba, Jordan, was probably a church,谓语动词应用单数的 was,而不是 were。
D 选项:indicates 应该用被动语态,而不是主动语态。这是因为,“一个已毁坏
的结构是一座教堂”这个事件肯定是“被”计划和朝向表明的,而不是主动表明了
什么。
E 选项:正确,That a ruined structure found at Aqaba, Jordan, was probably a church
为主语从句,“一个已毁坏的结构可能是一座教堂”由...表明。

994

免费加入GMAT备考群、机经群,可加微信:jmtccca
免费领取GMAT资料、了解GMAT课程,可加微信:jmtccca

In the major cities of industrialized countries at the end of the nineteenth century,important
public places such as theaters,restaurants,shops,and banks had installed electric
lighting ,but electricity was in less than 1 percent of homes,where lighting was still provided
mainly by candles or gas.

A.electricity was in less than one percent of homes, where lighting was still

B.electricity was in less than one percent of homes and lighting still

C.there were less than one percent of homes with electricity, where lighting was still being

D.there was less than one percent of homes that had electricity, having lighting that was still

E.less than one percent of homes had electricity, where lighting had still been

解析:A 选项:正确。
B 选项:逻辑上两句话不构成并列关系,是主次修饰关系,且 and 后的句子无完
整谓语动词。
C 选项:where 引导的定从就近修饰 electricity 错误。
D 选项:在逻辑上,我们不能说“房子拥有光亮(home have lighting)“,只能说”
房子里有光“,因为房子不能主动的拥有光亮。
E 选项:where 定从错同 C;房子不能主动拥有光亮错同 D。

995
By 1999, astronomers had discovered 17 nearby stars that are orbited bv planets about the
size of Jupiter.

A.had discovered 17 nearby stars that are orbited by planets

B.had discovered 17 nearby stars with planets orbiting them that were

C.had discovered that there were 17 nearby stars that were orbited by planets

D.have discovered 17 nearby stars with planets orbiting them that are

E.have discovered that 17 nearby stars are orbited by planets

解析:A 选项:正确。注意时态,定语从句 that are orbited by planets about the size


of Jupiter 中的一般现在时是正确的,因为 17 颗星星被环绕是一个客观事实,这
个动作的时间不受 discover 的发生时间的影响。
B 选项:首先,with planets orbiting them 是一个独立主格结构,其应改为修饰 stars
的定语从句。这是因为,独立主格是状语,修饰的是 discover 这个句子。在逻辑
上,不能说“伴随着星星被环绕,科学家们发现了 17 颗星星”,而应该是“科学家

免费加入GMAT备考群、机经群,可加微信:jmtccca
免费领取GMAT资料、了解GMAT课程,可加微信:jmtccca

发现了 17 颗被环绕的星星”。且 that were 这里是过去时,意思变成这些行星“过


去”和木星一样大,但现在不一样了,这是不合逻辑的,应该用一般现在时表示
一个客观事实。Them 指代歧义。
C 选项:时态错同(B)。
D 选项:by 1999,过去的过去,要用过去完成时;独立主格错同(B)。
E 选项: by 1999,过去的过去,要用过去完成时;discover 接宾语从句 that 17 nearby
stars are orbited by planets,核心语义有误,逻辑上,天文学家发现的肯定是星星,
而不是星星周边的事情。

996
Although she was considered among her contemporaries to be the better poet than her
husband, later Elizabeth Barrett Browning was overshadowed by his success.
A.Although she was considered among her contemporaries to be the better poet than
her husband, later Elizabeth Barrett Browning was overshadowed by his success.

B.Although Elizabeth Barrett Browning was considered among her contemporaries


as a better poet than her husband, she was later overshadowed by his success.

C.Later overshadowed by the success of her husband, Elizabeth Barrett Browning’s


poetry had been considered among her contemporaries to be better than that of her
husband.

D.Although Elizabeth Barrett Browning’s success was later overshadowed by that of


her husband, among her contemporaries she was considered the better poet.

E.Elizabeth Barrett Browning’s poetry was considered among her contemporaries as


better than her husband, but her success was later overshadowed by his.

解析:she was later overshadowed by his success.不是人和成功比,是成功和成功


比,Elizabeth Barrett Browning‘s success was later overshadowed by that of her
husband,that 指代 success,排除 ABC 项;D 选项,正确;E 选项,poetry 和 husband
不可比。

997
In no other historical sighting did Halley’s Comet cause such a worldwide sensation as did its
return in 1910–1911.

A.did its return in 1910–1911

B.had its 1910–1911 return

C.in its return of 1910–1911

D.its return of 1910–1911 did

免费加入GMAT备考群、机经群,可加微信:jmtccca
免费领取GMAT资料、了解GMAT课程,可加微信:jmtccca

E.its return in 1910–1911

解析:这题比较的是状语。A 选项,Halley's Comet 和 its return in 1910-1911.对


比错误;B 选项错同 A,就是 did 换成了 had。
C 选项,正确。
正常语序后整句话为“ Halley's Comet caused such a worldwide sensation in no
other historical sighting as (Halley's Comet caused) in its return of 1910-1911”括号中
的主谓结构是比较结构中的第二项,由于和第一项内容基本相同,所以被省略了。
所以本题实际上比较的是“哈雷彗星”在“other historical sighting(其他历史环境
下)”和“哈雷彗星”在“它 1910-1911 的回归地球”的历史环境下,其造成的“轰动”
的比较,“In no other historical sighting”和“in its return of 1910-1911”分别参与比较
的两个主谓结构的时间状语。其他选项中“its return in 1910-1911”要么在和
“Halley's Comet”直接比较(排除(A)、(B)、(D)),要么不知和谁在比(排除(E),没
能建立准确合理的比较关系。
语义:在其他任何历史情境下,哈雷彗星造成的全球轰动也比不上它在 1910-1911
年的回归中造成的轰动。

998
Rock samples taken from the remains of an asteroid about twice the size of the 6-mile-wide
asteroid that eradicated the dinosaurs has been dated to be 3.47 billion years old and thus
is evidence of the earliest known asteroid impact on Earth.

A.has been dated to be 3.47 billion years old and thus is

B.has been dated at 3.47 billion years old and thus

C.have been dated to be 3.47 billion years old and thus are

D.have been dated as being 3.47 billion years old and thus

E.have been dated at 3.47 billion years old and thus are

解析:A 选项:主语是 rock samples 主谓单复数不一致;且 be dated to be 错误,


只能用 be dated to 或 be dated at;将 date 换为主动语态则有:X dated samples to be
3.47 billion years old.
B 选项:主谓单复数不一致;and 后接的句子缺少谓语动词。
C 选项:be dated to be 错同(A)。
D 选项:and 错同(B)。
E 选项:正确。

免费加入GMAT备考群、机经群,可加微信:jmtccca

You might also like